Download as docx, pdf, or txt
Download as docx, pdf, or txt
You are on page 1of 168

18.5.

2018 Taiwan center 11:30 am

1. New question: family of an old lady with dementia complaints you


about they can no longer taking care of her. She awaked early in the
morning and go through the drawers and garbage. Often accuse the
family as thieves. What is the management?
A-urgently arrange for the nursing home care
B-arrange community nurse to see them
C-admit the patient
(no option for hospice care)
AAAAAAAAAAAAAAAAAAAAAAAAAAAAAAA

2. New Question: An 78 year old man came with acute confusional state
for 3 days. He had diabetes. He had an ulcer in medial mellious for 6
months or 6 weeks taking care by community nurse. There is long
description of ulcer like edge and margin. What investigation?
A-Blood culture
B-wound swab
C-CT head
D-Lumber puncture
BBBBBBBBBBBBBBBBBBB……….the ulcer looks to be infected

3. New Question: an old lady came with confusion after surgery of


removal of glioma.
There is description of Na-125, Cl-95, K 5.5, Glucose 8 and other lab
values.
Which of the following should be done to get diagnosis?
A-Serum osmolarity
B-Urine osmolarity
AAAAAAAAAAAAAAAAAAAA……………….SIADH

4. New Question: an old man came with confusion for 3 days. There is
description of Na-125, Cl-95, K5.5, Serum osmolarity-291, Urine
osmolarity-365 (not so sure about the osmolarity value). The patient
is well hydrated. What management should be given?
A-fluid restriction with 1.5L for 24 hours
B-Give Saline 0.9 1L for 6 hours
C-Give dextrose
CCCCCCCCCCCCCCCCCC??? ….normal osmolarity + confusion

5. New question: An 78 years old plumber man with 80 packs year


smoking came with progressive dyspnea. He didn’t relieved with
salbutamol, FVC 50% in 1 L and 70% in 2.5L, asking investigation? X
ray given.
A-CT chest
B-oral steroid
C-pleural biopsy
If x-ray shows pleural effusion then AAAAAAAAAAA then
CCCCCCC…..mesothelioma

6. New question: A general practitioner want to know the incidence of


hepatitis C in a community. Which of the following is correct?
A-All patient with hepatitis C antibody positive
B-All patient with hepatitis C PCR positive
C-All patient with hepatitis antibody in the past 5 years
BBBBBBBBBBBBBBBBBBB…………incidence = new cases

7. New Question: An old lady presents with lethargy and nausea not
vomiting. She had diarrhea for a few days. She had normal abdominal,
chest examination. Which of the following should be done?
A-FOBT
B-Colonoscopy
C-CT head
B BBBBBBBBBBBBBBBBBBBB…………colon cancer…..donot do FOBT in
high risk

8. New question: 24 years old 5th generation Australian father presents


you with 3 weeks history of dry cough. He didn’t had travel history. He
had an infant at home. What of the following is appropriate?
A-pertussive serology
B-blood culture
AAAAAAAAAAAAAAAAAAAAAAAAAAAA

9. A lady who had diabetes, hypertension, heart failure and on many


drugs complaints of dyspnea, orthopnea and bilateral oedema up to
knee. She fails to take drugs for 3 weeks during trips. After doing xray
and ECG, which of the following should be done for the management?
A-serum electrolytes
B-ABG
C-LFT
BBBBBBBBBBBBBBBBBBBBBBBBBBB

10. New Question: CT scan given showing left sided RCC. The patient
came with acute onset of pain in the RHC. Pain subsided in 3 hours,
no other abnormal findings. The abdominal examination is normal.
Which is the following is correct?
A-Gall stone
B-haemorrhage in to the adrenal gland
C-Ca head of pancreas
D-RCC
AAAAAAAAAAAAAAAAAAAAAAAAAAAAA

11. 55 yrs old male, known case of haemochromatosis not using IV


Drugs since last 5 yrs. Alcoholic. On presentation, he is drowsy and
hepatic flap. O/E gynecomastia, fever 38’C, mild ascites, spider nevi.
What is the most initial investigation?
A. FBC
B. Ammonia
C. Abd. Paracentesis
D. LFT
E. IV antibiotics
CCCCCCCCCCCCCCCCCCCCCCCCCCCC

12. New question: 25 years old nulliparous presents with nausea,


lethargy but no vomiting. Otherwise normal. She is currently 18 weeks
pregnant. Which of the following investigation should be done?
A-FBC
B-iron study
AAAAAA??...........BETTER H-CG

13. ] A man with depression on venlafaxine for 4 weeks. He is now


well controlled. But sometimes he see dark shadow when he is about
going to sleep. How to management?
A-he should be on anti-psychotic
B-tell him normal people also see dark shadow
C-he should change anti-depressant
D-he should cease venlafaxine
BBBBBBBBBBBBBBBBBBBBBBBBBBBBB

14. ] a lady with eating disorder during childhood. Now she


sometimes skip breakfast and eat a large meals in lunch and skip
dinner. She denies having mood symptoms. What is your advice?
A-advise her to eat small and frequent meals
B-it’s ok to skip meal if she eats healthy food
C-should have healthy life style and regular meals
CCCCCCCCCCCCCCCCCCCCCCCCCCCCC

15. 3 years old child with recurrent upper respiratory tract infection
for every 4 to 6 weeks for 6 to 8 times in a year. What is correct? (short
scenario, not mention about failure to thrive)
A-didn’t complete the antibiotic course
B-immunodeficiency
BBBB ???

16. ] a lady with palpitation, weight loss 5 kg, sweating. On


examination, fine tremor, warm hand, sweating. On lab, TSH is
decreased. Which of the following finding should be find?
A-High T3 and T4
B-diffuse uptake of the glands on scan
C-antibody positive (sorry can’t recall the name)
………….all right

17. 70 yr old woman with CA pancreas scenario with obstructive


jaundice. EF 20% what to do?CT scan given.
A-Cholecystectomy
B-Choledocojeujenostomy
C-Whipple
D-Mrcp
E-Ercp
EEEEEEEEEEEEEEEEEEEEEEEEEEEEEE

18. Pregnant lady with Hepatitis C comes to you for advice. Which
will reduce the risk of transmission to her baby?
A. C section
B. Avoid Breastfeeding
C. fetal heart rate monitoring with scalp clip is avoided (exact word)
D. Baby should be screened between 12 and 18 years of age to
determine whether they have been infected
E. Coexistance of HIV has no effect on HCV transmission
CCCCCCCCCCCCCCCCCCCCCCCCCCCCC

19. You are a doctor in a town, where 6 people out of 100 are non-
smoker get stroke. The chances of stroke in smoker is 50%more than
non-smoker. Now
the pharmaceutical company is introducing a medicine which reduces
the chances ofstroke by 1/3rd in smoker population. What is the
percentage of the stroke population will get stroke with treatment
using a new medicine?
A. 3%
B. 6%
C. 9%
D. 12%
E. 20%
BBBBBBBBBBBBBBBBBBBBBBBBBBB

20. A study wish to make a relation btw the fatigue in track car
driver in high ways & theincidence of MVA happen. wt is the most
suitable method to carry out study?
a. Cohort
b. Case control
c. RCT
d. Cross sectional
e. Case study
BBBBBBBBBBBBBBBBBBBBBBBBBB

21. One medical student is  studying relationship between amelgam


n dementia what type of study?
A. case control
B. cohort
C. cross sectional
D. RCT
E. Observational study
F. AAAAAAAAAAAAAAAAAAAAAAAAAAAAAAAAAA
G.
22. pharmacology company approaches u, claims new drug reduces
risk of stroke in AF patients by 40%, and has less GI side effects. Most
imp thing to ask regarding the information given?
a) length of follow up of patient after the trial
b) common side effects
c) absolute risk reduction
d) no of study participants
BBBBBBBBBBBBBBBBBBBBBBBBBBBBB

23. women with 2 years after Filshie clip sterilization. Regular cycle
with menorrhagia +, At this cycle, bleeding for 10 days with few clots.
Pap smear normal 12 months ago. Initial investigation? (exact option)
a) pap smear
b) Full blood count
c) ultrasound
d) Thyroid function test
e) Endometrial Biopsy
DDDDDDDDDDDDDDDDDDDDDDDDDDDDD

Dealing with excessive menstrual bleeding:

First step………..TSH

If normal………..TVS

If normal………look for other systemic causes


24. Indian university student, 2 months dysuria, hematuria,
frequency. All urine test normal except RBCs and WBC ++ in urine.
Urine culture (-). Dx?

a) Cystoscopy

b) Renal biopsy
c) Urogram
d) Repeat urinalysis
e) Chlamydia PCR

EEEEEEEEEEEEEEEEEEEEEEEEEE

25. Man with lack of interest in normal life, insomnia + and is having
visual hallucination and voices in his head +, he is saying my mother
died last month but it is not the case and mother died year ago, what is
most likely diagnosis?
A. Personality Disorder
B. Depression with unresolved grief 
C. Major depression with psychosis
D. Schizophrenia
DDDDDDDDDDDDDDDDDDDDDDDDDDD….grief is accepted up to 6
months 

26. A woman after removal of central venous line lady developed


facial swelling and swelling around the neck. What’s the most
appropriate investigation?
a. CT chest
b. CT neck angiogram
c. Chest x-ray
d. Echo
(no option for neck USG Doppler)
AAAAAAAAAAAAAAAAAAAAAAA……….ct chest with contrast

27. Gout scenario with chronic renal failure and ccf, now presents with
acute symptoms, what to give? (no prednisolone in option)
a) paracetamol
b) codeine 
c) naproxen
d) allopurinol 
e) prednisone
EEEEEEEEEEEEEEEEEEEEEEEE
28. patient has acout gout with no h/o chronic renal failure and prev
h/0 gout. what should be given-
a.allopurinol
b.naproxen
c.colchicine
d.ibuprofen
e. prednisolone
BBBBBBBBBBBBBBBBBBBBBBBBBB

29. An 18 year old girl presents with epistaxis, malaise and tiredness for
months. no other symptoms . Hb is 8.5, microcytic hypochromic
picture. INR is 1.5. Calcium is 1.9. What is next most appropriate to
reach diagnosis?
a. Hb electrophoresis
b. serum electrolytes
c. Iron studies
d. anti glidian antibods
e. stool culture
DDDDDDDDDDDDDDDDDDDDDDDDD…..celiac

30. Lawyer who is a perfectionist, recently fall in performance after


miscarriage, affect sleeping, crying, can’t concentrate her work and
problem with husband. don’t want to take medicine coz it might be
habit forming. what is suitable management at this stage?
a) CBT
b) SSRI
c) Interpersonal therapy
d) Diazepam
AAAAAAAAAAAAAAAAAAAAAAAA

31. Man with severe depression treated with venlafaxine now


presented with agitation pressured speech euphoria and mania ask
what next appropriate thing to do
a. add sodium valproate.
B. add clozapine
c. add olanzapine
d. Withdraw venlafaxine 
e. add risperidone
DDDDDDDDDDDDDDDDDDDD then CCCCCCCCCCCCC………….bipolar

32.CT scan given, chest MVA with severe chest pain and difficulty breathing
due to pain. Tenderness present on both side of chest. Decreased breath
sounds bilaterally and dullness to percussion. Asked about the cause of
pain. Diagnosis? 
a) Fractured ribs
b) Hemothorax
c) Pneumothorax
d) Pneumomediastinum
AAAAAAAAAAAAAAAAAAAAAAAAAA

33. MVA scenario. Patient difficulty in breathing due to pain. Left sided
chest pain present. On examination, left sided breath sound diminished.
On breathing, left side is preceded by breathing. On xray, there is no
pneumothorax. Which of the following should be done?
A-CT chest and abdomen
B-intubation and positive pressure ventilation
C-chest drain
d. morphine
DDDDDDDDDDDDDDDDDDDDDDDDDDD

34. Pt comes with haematuria CT given showing RCC asked what will
increase the chance of DVT after his operation?
a. Nicotine stain of fingers 
b. Atrial fibrillation 
c. Bilateral varicose veins
d. BMI 
e. Spider naevi on chest
DDDDDDDDDDDDDDDDDDDDDDDDDDDDDDDDDD

35. female came with symptoms fever myalgia, rash,, 1week ago,
went camping in Queensland, got bitten by tick, bite site got red and
tick was removed. o/e axillary lymphadenopathy , black scab at bite
site,etc. Dx?
a) Lyme disease
b) Scrub typhus
c) Q fever
d) ross river fever
BBBBBBBBBBBBBBBBBBBBBBBBB

36. adhd patient developing symptoms of mania, drink coffee and


energy drink during gambling
Excessive gambling.. Insomnia.. Spending money in pathological
gambling.Options were
Urine drugs screen
Blood alcohol
AAAA???

37. Patient got operated for appendicectomy. There was carcinoid


incidentally detected which was resected along with appendicectomy.
Persistent diarrhea present ever since preoperative period. No other
symptoms of carcinoid.
A-loperamide
B-octreotide
AAAAAAAAAAAAAAAAAAAAA…………then BBBBBBBBBBBBB

38. Old patient with decrease in vision from past few months, vision
6/18 on right side and 6/12 on left and on correction with pinhole is
6/9 on both sides. What is the cause from this condition?
A- Retinal degeneration
B- Glaucoma
C- Cataract
D- presbyopia
CCCCCCCCCCCCCCCCCCCCCCCCCCCCC

39. Old male with prostate cancer T3N0M1 invasion in seminal


vesicle and lumbar vertebra asked Mx?
A. Bilateral orchidectomy
B. External beam radiotherapy
C. Androgen deprivation therapy
D. Active surveillance
E. CCCCCCCCCCCCCCCCCCCCCCCCCCCCCCC

40. A lady was in Operation for hours and stayed in lithotomy


position ( mentioned exactly). After that she suffered of foot drop,
where the lesion?
A. S 1
B. neck of fibula
C. L 5
BBBBBBBBBBBBBBBBBBBBBBBBBBB

41. Lady comes on 10th postpartum day with pain due to perineal
tear.Now on examination there is perineal wound that is 2 cm long,1cm
wide and 1 mm deep.It was clean with no discharge.
What to do next?
A. Apply local antibiotics
B. Put on oral antibiotics
C. Simply keep the wound clean
D. Suture the wound under local anaesthesia
E. Suture the wound after cutting the edges
CCCCCCCCCCCCCCCCCCCCCCCCCCC

42. An old lady above 60 has history of breast cancer treated


surgically 3 years ago presents for osteoporosis screening her bone
density scan shows -2.7 how to treat it
a. Raloxifen
b. alendronate
c. vitamin d calcium
AAAAAAAAAAAAAAAAAAAAAAAAAA

43. 24year old man confined with a friend came mentioning “funny
business is going around" same in inverted comma) he had a feeling
something is going to happen. Euphoric state. (it was diff
scenario,nothing about promotion was mentioned) . What is the
example of his comments?

a) prodormal schezo
b) passivity phenomenon (delusion of control)
c) idea of reference
d) delusional disorder
AAAAAAAAAAAAAAAAAAAAAAAAA ?
44. a 10 yr old child had seizure at home.after that he became
unconscious and remain.he was brought to ed via ambulance.blood
glucose 2.wat will u give
a.im glucagon
b.iv glucagon
c. iv dextrose
d.phenytoin
e.diazepam
CCCCCCCCCCCCCCCCCCCCCCCCCCC

45. 5 yr old child with lacerated wound. Receives DTPa at 2 month


and 4 month after that no booster. What to give him now?
a. tt or tig
b. dtpa now and booster after two month
c. tt +topical antibiotic ointment
d. Dtpa n tig
BBBBBBBBBBBBBBBBBBBBBBBBBBB

46. 75 years old man presents with severe retrosternal chest pain.
Pain is preceded by vomiting, with BP 100/70 mmHg, dull on
percussion and reduced breath sound at the base of the lung. He has
HTN DM asking most diagnostic test?
A-non contrast CT
B-ECG
C-CXR
D-gastrograffin swallow
DDDDDDDDDDDDDDDDDDDDDDDDDD

47. Patient writes bad things about doctor on facebook. What to do:
A-Inform medical defense organization
B-Give feedback on the post and explain
C-Write to ask him to remove post
D-Ignore
e.talk to him
EEEEEEEEEEEEEEEEEEEEE

48.39 weeks pregnant lady came with labour pain. Synto was given
appropriately. CTG was done which shows heart rate of 140 which
dropped to 70 and came back to 140 in 2min. asking next appropriate
treatment. (Que does not include any basic measure given for her like
Left lateral position, Oxygen, Fluid) 
A. Stop syntocinon 
B. Fetal scalp sampling 
C. C section 
D. Give IV dextrose, normal saline
E. Give O2 to mother via face mask
AAAAAAAAAAAAAAAAAAAAAAAAAAAAA

49.Lady full term with adequate contraction was monitored on CTG.


Everything was progessing normally. Suddenly there was a
deceleration of FHR dropping to 70bpm for 4 mins. What is the next
most appropriate step?
a. Stop syntocinon
b. Continue synto at an increased dose
c. Immediate cesarean 
d. Ventouse delivery
AAAAAAAAAAAAAAAAAAAAAAA

50. A 6 month old child came with recurrent upper respiratory


infection. The immunization history reveals that the child has received
his hepatitis B vaccination at his birth and no further immunization
history at 2 and 4 month of age due to recurrent upper respiratory
infection. Now the child has fever 37.8 and clear nasal discharge. What
is your management regarding immunization?
A. Give hepatitis B immunization and institute catch up schedule
B. Give hepatitis B immunization and recommence standard schedule
C. Investigate underlying cause of URTI
D. Give hepatitis B immunization after fever subside
E. Give hepatitis B immunization at now
AAAAAAAAAAAAAAAAAAAAAAAAAAAA

51.2y.o. Boy vomits after feeding, cries, lost 200g in previous week. Blood
analysis shows metabolic alkalosis. What is the investigation?
A) CT
B) Barium enema
C) Ultrasonography
D) Endoscopy

CCCCCCCCCCCCCCCCCCCCCCCCCC
52. Then patient concerned Abt wgt gain on antipsychotic now
balanced life on olanzapine
Option was

A. to stop olanzapine

B. Change to ziprasodone
BBBBBBBBBBBBBBBBBBBBBB

C.

53.18 m Child crying and waking up at night everytime mother give fruit
juice .Ur advice
A.add milk thickener to bottle
B.give cows milk
C.give water.
D.give bottle milk
AAAAAAAAAAAAAAAAAAAAAAAAAAAAA

54.Fournier gangrene photo


Asking management

A. Debridement

B. Hyperbaric o2

C. Antibiotics

AAAAAAAAAAAAAAAAAAAAAAAAAAAAA

55.A man with peptic ulcer who is taking triple therapy (PPI, amoxicillin,
metronidazole) for 6 weeks but urease breath test was positive for
H.pylori
what is the reason?
a. resistance to metronidazole
b. resistance to amoxicillin
c.unreliable urea breath test
AAAAAAAAAAAAAAAAAAAAAAAAAAAAAAAA
56. Previously healthy woman came for AN 38 wk gestation with 1L
blood loss. Low lying placenta seen during 18 week care and failed to
follow up and USG check. On exam: bp 100/70, Hb - slightly low.
Which further process will help u for dx?
A. Pain with bleeding
B. Uterus TENSENESS
C. Absence fetal heart sound
D. Uterine contraction
E. High head
EEEEEEEEEEEEEEEEEEEEEEEEE

57. Pacemaker syndrome with difficulty in breathing with faint heart


sounds,spo2 96%, bp-90/60mmhg,jvp-6 with no ECG given.
investigation of choice needed.
A. Chest xray.
B. CTPA. 
C. ECG
D. ECHOCARDIOGRAM
E. SERUM TROPONIN
DDDDDDDDDDDDDDDDDD…………tamponade

If next ……………AAAAAAAAAAAAAAAAAAAAAAAA

58. 35 year old pregnant woman presented at 15 weeks gestation


asking you for a diagnostic test of down, what is the diagnostic test you
should do for this woman? 
A-nuchal translucency ultrasound 
B-maternal serum and ultrasound 
C-maternal serum 
D-amniocentesis 
E-chorionic villous sampling
DDDDDDDDDDDDDDDDDDDDDDDD

59. 12 week old pregnant lady was brought to the emergency


department with vomiting, abdominal pain and severe bleeding for the
past 4 hours. She also had 2 fainting attacks. Her blood pressure is
80/40.She was normal till before this episode and USG at 11 weeks
showed normal nuchal translucency. Which is the most likely
diagnosis?
Incomplete abortion
Complete abortion
Abruption placenta
Ectopic pregnancy rupture
Hydatidiform mole
AAAAAAAAAAAAAAAAAAAAAAAAAAA

60. past history of dvt pt on UFH perioopertively n switch to lmw


heparin after 5 days undergone rt hip surgery develop dvt after 10
days that what to do
In investigation only platelets r decreased

A- ffps
B- vit k
C- cease heparin and switch to other anticoagulant
E- platelets infusion
CCCCCCCCCCCCCCCCCCCCCCCCCC

61. 72 years old Man come with low back pain. On investigating,
bone scan shows increased uptake in L4-L5. Many lab investigations:
low Hb around 85 , mild leukocytosis , ESR 108, What will you do to
know the cause of fracture ?
A. PSA
B. Bone marrow examination
C. MRI spine
D. Spine X ray
BBBBBBBBBBBBBBBBBBBBBBBB

62. Boy 7 years old with recurrent abscesses. On examination.


Pharyngitis, gingivitis and multiple scabs of abscesses. Low weight and
height. What investigation is helpful for the diagnosis?
A-lymphocyte count
B-neutrophil function
C-complement level
D-antibody level
BBBBBBBBBBBBBBBBBBBBBBBBBBBB……….CGD

63. A man with difficulty in climbing stairs and difficulty in standing


from sitting. One week ago, he had diarrhea. No sensory loss. What
investigation?
A-LP
B-EMG
C-spirometry
AAAAAAAAAAAAAAAAAA…………..GB$
64. Patient complaining of loss of sensation over lateral arm, lateral
forearm, loss of triceps jerk, asking about the site of injury.
A-C5, 6
B-C6, 7
C-C8, T1
BBBBBBBBBBBBBBBBBBBBBBBBBBBBB

65. Patient I forget the scenario but he has loss of planter flexion
and inversion, but he has intact dorsiflexion and eversion. Knee jerk
normal. Which nerve injury?
A-sciatic
B-tibial
C-common peroneal
BBBBBBBBBBBBBBBBBBBBBB…………..TIP

66. A mother came with her child complaining of continuous


bleeding after fall from coffee table. 
On exam there is MINIMAL bleeding from the frenulum and child had
some old bruises on forehead and legs. there was no petechiae,
lymphadenopathy and hepatomegaly. What is the most likely cause
a. Idiopathic Thrombocytopenic Purpura 
b. non accidental injury
c. VWD 
d. clotting defect
BBBBBBBBBBBBBBBBBBBBBBBBBBBBB

67. 24yrs old with dysmenorrhoea and menorrhagia asked to do


investigation but she insist she wants medication 
a. mefenamic acid during the period
b. tranexamic acid during the period
c. ocp 
d. iucd
BBBBBBBBBBBBBBBBBBBBBBBBBB

68. An old woman presents with painless haematuria. On CT, there is


5 cm mass in the right kidney.
Which of the following should be done?
A-surgical exploration
B-IVP
AAAAAAAAAAAAAAAAAAAAAAAAAAAAA

69. a pt has alzhemiers dementia and ca colon. U have planned


colostomy. One of her daughter asked as her mother is not capable of
caring and she don’t want her mother to be operate.her MMS is 20/30.
You talk to pt and pt has given consent for operation. What is your
most appropriate advice?
a) you will proceed for operate as pt has given consent
b) you will not operate as pt is not capable of giving consent
c) discuss with family physician and surgeon regarding management of
pt.
d) as her daughter has power of attorney she can give consent for not
operating her mother
DDDDDDDDDDDDDDDDDDDDDDDD

70. Old woman with urgency and nocturia for a long time. Now
having incontinence for like a month. She also has bilateral knee OA.
What is the best management for her? 
A. Fluid restriction at night
B. Install a commode in her bedroom. 
C. Oxybutynin 
D. Paracetamol
CCCCCCCCCCCCCCCCCCCCCCC………..urge

71. Aboroginal boy, his father died 1 year ago but he says he died 6
months back, is having hallucination and withdrawn from society, Dx:
Grief
Unresolved grief with depression
Major Depressive with psychosis
Schizophrenia
DDDDDDDDDDDDDDDDDDDDDDDDD

72. Man on vancomycin. He starts to develop a rash following the


infusion ofVancomycin. On stopping the drug the rash starts to go
down. Which of the following is the appropriate action?
a. Stop vancomycin
b. Slow the infusion rate for vancomycin
c. Give prednisolone
BBBBBBBBBBBBBBBBBBBBBBBBBBBB

73. decrease breast milk production recall Asking common cause


of decreased milk production in breastfeeding
a.Less frequent feeding.
b.Dec intake of fluids
c.Spend less time while baby is sucking
AAAAAAAAAAAAAAAAAAAAAAAA

74. mother came with her two children. Mother's brother has
haemochromatosis. What screening test advice would you offer?
a) Screen only mother
b) Screen mother and kids
c) Ask her to come with her husband
d) Screen only the children
CCCCCCCCCCCCCCCCCCCCCCCCCCCC

75. pt has colles fracture treated 2 months ago. Her xray at follow up
at 6 weeks was normal. Now c/o pain and tingling sensetions at the site
of fracture what u will do next?
1. repeat xray
2.Mri to see ligaments
3 exercises
4. occupational therapy to check on him
5.brace
DDDDDDDDDDDDDDDDDDD

76. a pt came for advice as her mother has fracture of hip due to
osteoporosis. Pt ask for proplylactic treatment as she is at risk of
having osteoporosis ,works long hours your next step in her
investigation?
a) serum ca and phosphate level
b) vit d level
c) xray
d) bone scan
DDDDDDDDDDDDDDDDDDDDDDDDDDDDD

77. A patient presented with macroscopic hematuria. 6 months back


he diagnosed as c-ANCA positive vasculitis causing hematuria and
started on Prednisolone and Cysclophosphamide. The RBCs on
examination are 20% dysmorphic and 80% normal, what is the most
appropriate next step?
a. Repeat renal biopsy
b. Stop cyclophosphamide
c. CT abdomen
d. Renal USG
e. Cystoscopy
BBBBBBBBBBBBBBBBBBBBBBBB

78.Pic of sle rash face and arthritis and recurrent mouth ulcers for 6
months.
ANA and Anti dsDna positive. Long term management?
Methotrexate
Hydroxychloroquine
Cyclosporin
BBBBBBBBBBBBBBBBBBBBBBBBBBBB

79.Pic of red eyes with recurrent oral and genital ulcers. Asking
management?
A-Oral steroid
B-topical steroid
AAAAAAAAAAAAAAAAAAAAAAAAAA

80. A man after surgery of cataract complaints of decreased vision


and red eyes. Pic of hypopyon.Diagnosis?
A-Hyphaema
B-hypopyon
BBBBBBBBBBBBBBBBBBBBBBBBBB

81. Patient with 10 minutes weakness in the body, no vision loss


A. Lacunar infarct
B. Carotid stenosis
AAAAAAAAAAAAAAAAAAAAAAAA

82. 36 wk pregnant female with normal ANC presents with fetal


bradycardia.Em LSCS done .Neonate has still have HR of 60.Whcich
antibodies contribure:
a)Anti Ro
b)Anti Sm
c)Anti cardiolipin
d)Anti nuclear antibody
AAAAAAAAAAAAAAAAAAAAAAA
83. A 4 year old girl is brought by her father complaining that she
has put on lot of weight in the past one week. On examination you find
edema on legs and peri orbital puffiness.. Which of the following will
lead you to diagnosis?
A-urine analysis
B-BP
C-Urine micro & culture
D-blood culture
AAAAAAAAAAAAAAAAAAAAAAAAAAAAAAA

84.Post hysterectomy patient with history of DVT post operative after


hysterectomy. Now comes with hot flushes. Which HRT is best?
Low dose Oral estradiol
Oral estrogen plus Progesterone
Oral Progesterone alone
Transdermal estrogen therapy(exact words)
Transdermal estrogen and progesterone therapy
DDDDDDDDDDDDDDDDDDDDD

85.51 years old woman had her last menstruation 8 months back. She is
complaining of host flushes and insomnia. Which preparation is best for
her? 
a) Continuous HRT with oestrogen and 12 days progeterone 
b) Continuous HRT with oestrogen and progesterone
c) OCP 
d) Oestrogen patches
AAAAAAAAAAAAAAAAAAAAAAAAAAAAA

86. 7 days old infant presented to you complained with poor feeing and
bile stain vomiting. His birth was at 38 weeks gestation and weight
2600g. He is breastfeeding with no immediate post natal complications.
The child has a history of passing meconium on day 4 of birth. Now, he
had mild jaundice and abdominal distension present. What is the
appropriate diagnosis?
a) Hirschsprung's Disease
b) Meconium Ileus
c) Duodenal Atresia
d) Necrotizing Enterocolitis
e) Volvulus
AAAAAAAAAAAAAAAAAAAAAAAAAAA

87. Farmer and his wife has 3/12 baby. wife is tearful, fearful for
child. you are a rural GP.
A. start sertraline
B. bring child to protection service
C. admit mom to hospital
AA?? Needs whole scenario

88. Patient with anorexia nervosa. Which clinical finding?


A-ankle oedema
B-vitiligo
C-tachycardia
(no option for lanugo hair)
AAAAAAAAAAAAAAAAAAAAAAAAAAAAA

89. 24 months old child presents for well child check. Which
language milestone should he achieved?
A-2 or more personal pronouns
B-know 4 colors
C-know family name

90. 3 years old child with macular generalized rash with cracked
lips oral mucosa involved
bleeding from hand feet genitalia
Labs thrombocytosis
And leucocytosis

Most appropriate asked--


Echo
Blood culture
Urine culture and pcr
AAAAAAAAAAAAAAAAAAAAAAAAA………..kawasaki

91.Surgeon shouting, some staffs cry. You (an intern) should do?
A. talk to surgeon,
B. talk to anesthesist,
C. raise the problem at the intern meeting,
D. report to director of clinical training
DDDDDDDDDDDDDDDDDDDDDD

92.Human bite recall. 35 years old man went to pub, where a woman bite
him, totally unknown and she went away. 2 cm lesion, bleeding
continues, teeth mark, he is fully tetanus immunized 3 years ago. What
to do now?
A-suture and review in 24 hours
B-hep B vaccine and immunoglobulin
C-zidovudin
D-flucloxacillin
E-Aziathromycin
BBBBBBBBBBBBBBBBBBBBBB

93. Intern at the hospital. He knows about all the needle stick injury.
All cases in the hospital were managed correctly with no big problem.
Now there is accreditation and the intern were asked about this issue.
What the intern should answer?
A-give info about the protocol of management
B-leave the answer to your supervisor
C-tell a case that he knows
D-say that there’s no problem with needle stick injuries in this hospital
AAAAAAAAAAAAAAAAAAAAAAAAAAAAA

94. Picture given of facial palsy and deep jaundice of eyes. The old
lady presents with facial palsy for one day duration. She also had
painful vesicle in the ear the day before. What is the management?
A-prednisolone
B-acyclovir
BBBBBBBBBBBBBBBBBBBBBBB………RAMSAY HUNT $

95. A man comes with weakness of lower limbs. She drags her foot
while walking, dorsiflexion and planter flexion is weak. Reflexes
are exaggerated. Equivocal planter response.What is the diagnosis?
A-motor neurone disease
B-cervical spine MS
C-spinal canal stenosis
D-lesion in cerebral cortex
CCCCCCCCCCCCCCCCCCCC………..lower limb only affected
96. Patient who has HTN with DM & AF on metformin, Statin and
started amiodarone 2 weeks earlier suddenly got swelling at right
thigh for 12hours which is progressively increasing and become
painful and got fever 37.8.Right thigh size is 4 cm larger than left.
Diagnosis?
A)Rhabdomylosis
B)Hematoma
C)DVT
D)Cellulitis
E)drug interaction
CCCCCCCCCCCCCCCCCCCC

97. Old woman 75years is admitted to the hospital following a


community acquired pneumonia. She received antibiotics and was
recovering. on the 4th she suddenly develops rigors,high grade
fever,bibasal crackles was mentioned only on examination.What is the
most likely diagnosis?
A. Hosp acquired pneumonia
B. Iv cannula related bacteraemia
C. Empyema
D. Pulmonary embolism
AAAAAAAAAAAAAAAAAAAAAAAAAAAAAAAA

98. X ray with high hemidiaphragm from one side.He is a smoker and
has a history of chronic cough now.He has 2 weeks history of rusty
sputum Diagnosis?
A. Bronchiectasis
B. Pulmonary Fibrosis
C. COPD
D. Acute on chronic bronchitis
DDDDDDDDDDDDDDDDDDDDDDD

99. Pt with DM & HTN, so on ACEI, Diuretics, metformin, statin


&amitryptylline for neuralgic pain. Planning for elective ortho surgery
in next wk, asking which drug to stop one wk before Sx?
A. Statin
B. Peridopril
C. Indapamide
D. Metformin
E. Amitryptylline
EEEEEEEEEEEEEEE…………..NSAIDS, AMITRYPTILLINE,
WARFARIN ANTI PLATELETS…ONE WEEK BEFORE SURGERY
100. Picture of external thrombosed pile. What is the clinical finding?
A-pain + bleeding while defecation
B-pain + bleeding not related to defecation
C-no pain + bleeding while defecation
D-no pain + bleeding not related to defecation
E-pain but no bleeding
BBBBBBBBBBBBBBBBBBBBBBBBBBBBB…thrombosed

101. You r in intern doctor nurse call u,a 10%burn want to discharge
by
himself, he heard your conversion about him, he is very annoyed
something like that's, what is your next mx
1.tell the nurse u will come & see him
2,dischargehim&following him in as outdoor pt
3.inform burn registrar unit that pt want to discharge
AAAAAAAAAAAAAAAAAAAAAAAAAA

That’s all I remember.

70 yr old woman with ca pancreas scenario with obstructive jaundice EF 20% wht
to do ??
Cholecystectomy
Choledocojeujenostomy
Whipple
Mrcp
Ercp

EEEEEEEEEEEEEEEEEEEEEEEE

Case of community acquired pneumonia was treated and got well on day three
after giving 2 antibiotics (IV augmentin, oral roxithromycin) and given enoxaprin.
On day five patient started having fever 39F, chills and rigors associated with
crepitation in right lower zone. Foul smelling purulent white nasal discharge.
a. Empyema
b. Hospital acquired pneumonia
c. Drug hypersensitivity
d. Iv cannula bacteremia
e. Pulmonary embolism
BBBBBBBBBBBBBBBBBBBBBBBBBBB

10 weeks post partum with complaints of baby crying, baby is normal. What will
u check in hx.
…this q option were changed which I cant remember

Lots of prostate and back pain q..with bit changed option

A man aged 68 years. his psa is 3.8ng now. last 2 year( sure) it was 1.5.he did 12
biopsies this time and one showed a focus of adenocarcinoma and gleason score
4. he had operation for cardiac stent placement 18months back & on dual
antiplatelet since then.What is the management for this man?
A) radical prostatectomy
b) external beam radiotherapy
C) orchidectomy
D) continued surveillance

DDDDDDDDDDDDDDDDDDDDDDDDDDDD

A patient comes to reduce weight. What is the important factor for her obesity?
a. bmi
b. waist circumference
c. waist hip

BBBBBBBBBBBBBBBBBB

For health risk ………CCCCCCCCCCCC

The patient’s investigations are “ T score -2.5, normal serum calcium level, low
serum Vitamin D level and other lab investigations are within normal range.
What will you prescribe her?
Vitamin D
HRT
Oestrogen
Alendronate

AAAAAAAAAAAAAAAAAAAA………..correct vit D 1st


If normal vitamin D then …………DDDDDDDDDDDDDDD

Same q with pt having GERD ..2 q one q option was strontium and q zolindronic
acid

What is the best method of study for studying the relation between fatigue and
accident in truck driver?
a. RCT
b. Cohort
c. case-control study
d. Case report

CCCCCCCCCCCCCCCCCCCCC

What study will you do to find out association of dental amulgumsmurcury and
occurrence of dementia in your community
1.Case control
2.Cohort
3.RCT
4.Case study

AAAAAAAAAAAAAAAAAAAAAAAA

CT haemopnemothorax..asking cause of pain

Back pain..lots of q

Xray scaphoid fracture…mngment

A woman works at a part-time job, and complaints of early morning headache,


frontal & bilateral, dull in character, varying in intensity, she takes paracetamol &
ibuprofen, which only cause relief for 2-3 hours. What is the cause?
a. drug rebound headache
b. migraine
c. tension headache
d. cerebral tumor

CCCCCCCCCCCCCCCCCCCCC

35. Pt comes with H/O of unilateral temporal side headache for last 4 hours. He
didn’t experience such type of headache before. He is also complaining of mild
neck pain of same side. Pain is not subsiding by taking regular pain killers. Pic of
the pt is given below. What will be the next step in management?

a. CT head
b. ESR
c. Temporal artery biopsy
d. CXR
e. Visual evoke potential test

AAAAAAAAAAAAAAAAAAAAAAAA

Middle age man, pain in buttock and thigh during 100 m walk on ground, can
walk 20 m uphill, femoral pulse not palpable, dorsalis pedis is palpable, ABI 0.3.
Best way to diagnose?
A. Arterial Doppler
B. Digital subtraction arteriography
c.ct angiogram
d. arteriography

CCCCCCCCCCCCCCCCCCCCCCCCC

3. Elderly lady, presented with confusion, dizziness. She was on vacation for 3
months, her meds got over so didn’t take it for last 2 weeks. Was on multiple
drugs like Hydrochlorothiazide, perindopril & some more….. Labs given. Na:
130, K : 5.2, Glucose : 4 . Wat is the cause of her presentation?
A. Dehydration
B. Hyponatremia
C. Hypocalcemia
D. Hypoglycemia

………….addison disease……old recall

A male maybe 50 years with H/O renal transplant for 10 years. has fever,
headache, neck stiffness n rigidity. CXR explained in q as round opacity on
middle lobe. Dx?
a. TB
b. aspergillosis
c. lymphoma
d. pneumocystis pneumonia

AAAAAAAAAAAAAAAAAAAAAAAA

12.12 yrs old child who is taken from her parents bcoz they r drug addicts and
now living in foster,child plays wth her immaginary friend , good collection of
toys which she like toys and proud of ear same food always her carerdoenst like
tht what is the nost immediate danger she is in ?? 
A ) drug abuse
B) schizephreniform psychosis
C ) sexual abuse
D) depression 
E) anorexia nervosa

EEEEEEEEEEEEEEEEEEEE

58yrs old man, drag left foot, reflexes on left lower limb are increased, planter
flexion & dorsiflexion are 4/5, weak ankle movements, equivocal plantar reflex,
upper limb and face are normal exam.Lesion site
a. Common perineal nerve
b. Cervical spinal cord
c. L5,S1 nerve root
d. Cerebral cortex
e.Brain Stem

DDDDDDDDDDDDDDDDDDDDDDDDDD

adrenal mass 5cm find on CT scan other physical examination normal , what is
management
A - Surgery
B - Review after 6 month

…………none…….do hormonal evaluation 1st and if high biopsy


35 yr old lady presented to you for breast CA screening as one of her paternal
aunt is
with Breast and ovarian ca. What will u advice her?
A.2 yarely mammography
B.2 yarely ultrasound
C. conduct Genetic risk assesment
D. order BRCA1 gene mutation
E. 6 monthly self breast examinations.

………………ALL WRONG …………do mammography from age of 50 ys

https://www.racgp.org.au/your-practice/guidelines/redbook/9-early-detection-
of-cancers/93-breast-cancer/

34.patient presented with decompensated liver disease with decreased serum


albumin(2 ) wt to give

a. lactulose

b. spironolactone

c. fluid restriction

d. concentrated albumin

BBBBBBBBBBBBBBBBBBBBBBBBBBBBBBBBBBBB

..acute pancreatitis scenario.patient with sudden epigastricpain.when will u do ct


abdomen-

a.2nd day of admission

b.3rd day of admission

c.clinically when the condition deteriorates

d.now

CCCCCCCCCCCCCCCCCCCCCCCCC
a child 10 yr old refuses to go to school . he developed frequent touching of his
face and raise his arm above his head and crawl his hand on the ground, he can
supreess doing those thing s but developed anxiety if he suppress those things .
his classmates make fun of him coz of those behaviours and his teacher also
annoyed,what to give
a. Temazepam
b. Mirtazapine
c. Risperidone
d. Quitiapine

………….case of OCD………….give SSRI

.Mother living in mining town comes with her daughter for normal blood
test. Her daughter’s test show lead level 0.72 micromoles/L ; what will be
your next appropriate step?

A. Assess IQ testing of child

B.Educate mother about environmental risk

C. Refer for chelation therapy

D. Move the family to another town

E. Urine test to confirm dx

BBBBBBBBBBBBBBBBBBBBBB

66.Middle age man with HTN, DM was on


warfarin,perindopril,statin,metformin.1wk ago he started to take
amiodarone, now present with right thigh swelling. On examination temp
37.9, tender, red thigh. His Rt thigh is 4 cm bigger than left in
circumference. Cause of this presentation??

1.aterial embolus

2.Cellulitis

3.dvt

4.Drug reaction

CCCCCCCCCCCCCCCCCCCCC
.Young woman presents with epistaxis and fatigue , she felt of tiredness
for months and has had regular heavy periods. She is pale, otherwise in
no acute distress.On investigations her Hb is 85g/l (low)MCV (low)Ca is
1.9 (low) andINR is 1.5. (normal given with upper limit 1.1 so high INR)
Which of the following is the most appropriate next step?

A-blood transfusion

B-Iv calcium

C-hemolytic screen

D-fresh frozen plazma

E.Vitamin k

EEEEEEEEEEEEEEEEEEEEEEEE

woman comes to your clinic. She was prescribed trifluoperazine for her
condition. She was taking it for 3 years with improvement of her condition. She
says that she discontinued taking her medication for the last 3 weeks because
her doctor was not present for he was in trip, she

also said that trifluoperazine makes her hand or some muscle part stiffy, rigid,
and restless.Now she presents with voices in her head. What is the most
appropriate initial choice in management?

A-trifluoperazine

B-resperidone

C-quitapine

D-respa depot

E-stop trifluoperazine

CCCCCCCCCCCCCCCCCCCCCCCCCCCC

. A pregnant lady comes with h/o well controlled type 1 DM, she also had
mild renal impairment, her creatinine level was 120 micromole/l (normal
level 60-110 micromole/l). Which of the following risk will have in baby?
A. IUGR
B. Macrosomia
C. Renal agenesis 
D. Intrauterine fetal demise 
E. Low Birth weigh

BBBBBBBBBBBBBBBBBBBBBBBBBBBBt

Patient with COPD was admitted for 3 days in hospital. Bronchodilator was
given 8 hourly, ipratropium bromide, and oral prednisolone. Sputum was of
thick yellow colour. Despite the treatment, he still has severe respiratory
distress but he is alert. Arterial blood gas examination are as follow
pH - 7.35 (7.36-7.44)
PCO2 - 50 (30-45)
PO2 - 80 (80-100)
HCO3 - 35 (22-26)
What is the next appropriate step?
a. CPAP
b. bronchodilator 4 hourly 
c. change antibiotic
d. change to IV hydrocortisone 
e. arrange refer to ICU for intubation

BBBBBBBBBBBBBBBBBBBBBBBBBBBB

Sorry I couldn’t remember much.There are some new q but topics were
common and some qus with twisted options.so we should read the options
carefully.keep me in ur prayer.

18th May 2018-Sydney

Mother brings her 7 months old son with complaints of bleeding from mouth
and gums. she told you that he bumped his head somewhereso that he
presented likethis, on examination the child was bleeding from frenulum, old
bruises on shins and forehead. No hepatosplenomegaly. No petechial rash.
What is the most likely diagnosis?
�A-factor 8 deficiency
�B-non-accidental injury
�C-Von willebrand disease
�D-leukemia
BBBBBBBBBBBBBBBBBBBBBBBBBBBBBBB

2.Primigravidagave birth at term to 3200 g baby, normal vaginal delivery


without any complications and there was slight meconium staining of liquor.
Baby was normal at birth with normal heart rate. After 1 minute, suddenly
stopped breathing.HRdecreased. Pulse was 40/min. Cyanosed. No response on
stimulation. What is the most appropriate next step? �
3. a. Intubate �
4. b.Nasopharyngeal aspiration of meconium
5.�c. Oxygen by Bag and mask ventilation
CCCCCCCCCCCCCCCCCCCCCCCCCCCCCCCCC

6. What’s the most important markerin finding health risk of


obesity?
7.�- BMI
8.�- waist circumference

- waist hip ratio


- CCCCCCCCCCCCCCCCCCCCCCCC

9. Diabeticfoot ulcerwith pic.. asked what to do next

X ray

Venograp

hy MRI

AAAAAAA

10. One scenario with HF, raised JVP, dullness on lung percussion. On
digoxin, thiazides all lots of othermedication. ECGwas there with HF. Asked
what to do next. Digoxin levelwas 1.5 (normal was 2.5).

Increase digoxin dose

Stop all medications and review

Increase thiazide
CCCCCCCCCCCCCC

11. Renal stone at pelvis-2 cm-


ESWL
PSWL-
AAAAAAAAAAAAAAAAAAA

12. Young girls 9 years of age with history of recurrent URTI presented
aftersore throat. Urinalysis showed blood (+), protienuria(+). Now she came
afterone week. Urinalysis is unchanged except for Blood (+) which is non-
glomerularand there are no casts in the urine. Which of the following is the
next best investigation?

a.Urine culture

b.DMSA

c.Ultrasound

d.IV

AAA

13. 75 years old man, History of appendicetomy & cholecystectomy


presented with 3 wk history of abd pain, distension, BS exaggerated ,
rectum empty. CTabd image given
Sigmoidvolvulous

Adhesive IO

C- CA Sigmoid

D- Pseudo obstruction
CCCCCCCCCCCCCCCCCCCCCCCCCC

14. 2 min—stop syntocinon

15. 4 min---next-stop syntocinon


16. travelled asia. 2 weeks history of cough n mild fevern epigastricpain
but no rigidity,this same xray was there, cause of his presentation—didn’t
see any opacities orconsolidation

Tb

viral pneumonia

pneumoniapneumococ

al fungal infectjon

BBBBBBBBBBBBBBBBB

17. flial chest Q, CT given with rid fracture, MVA,#rib, painfull shallow
breathing ,O2 given by face mask,whats next?

a.morphi

nb.intuba

te c.drain

AAAAAAA

18. Child presented with UTI, on Ultrasound right kidney smallerin size
smallerthan left. What is the next investigation?

DMSA

Urine

culture CT

scan
AAAAA

19. 65-old woman presented with acute painfulred eye , on examination


therewas irregularreflex with halo and hazy cornea . what is the most
appropriate long term treatment forthis condition?

A-

iridotom

y B-

timolol

C-pilocarpine

D-acetazolamide
AAAAAAAAAAAAAAAAAAAAAAA

20. COPDscenario,lots of findings,comes with acute exaggerbation,co2


increase but o2 normal,no hypoxia,hypercapniapresent,antibioticgiven
already,whats next?

a.ventilation

b.cpap

c.change antibiotic
D. increase bronchodilator
Dddddddddddddddddddddddddddd

21. STAT Q---ASPIRIN Q---

Ans,100%

22. Old woman 75 years of age is admitted to the hospital following


acommunity acquired pneumonia. She received antibiotics and is well on the
5th day. She suddenly developed rigors, chills and high grade feveron the 4th
day. What is the most likely explanation?
a.Hospital acquired pneumonia

b.IV cannularelated bacteremia

c.empyema

d.Pulmonary embolism

AAAAAAAAAAAAAAAAAAAAAA

92-old patient found unconscious at home brought by son to emergency


department . the patient was taking tablets methadone, oxycodone ,
others , patient has constricted pupil on examination which of the
followings is the likely cause?

methadone

oxycodone

AAAAAAAAAAAAAAAAAAAAAAAAA

23. Pt with RA she e ,they mention the time it was years forboth of
drugs , pt complains of some symptoms , and herlabs are given ALT
,ASTGGT all were high , the Q is asking about the cause of this ?

Methotrexateinduced

hepatitis

B-Ibuprofen induced

hepatitis

C-Autoimmune

hepatitis

D- Viral hepatitis

AAAAAAAAAAAAA
24. Woman with prescription of HRT. She has been taking HRT for 6 years
now and no complaints. What will you do? Dexa-1.7

a.cease HRT

b.REDUCE HRT

c.CONTINUE HRT

d. AAAAAAAAAAAAA

25. young girl with dysmenorrhoe & menorrhagia---

Ans.mefenamicacid

26. Patient with decreased Hb with cyclical menorrhagia. Hysteroscopy


and Dand C done. Both normal. Wts next

a.levonorgestralreleasing IUCD

b.progesterone

from day 15-25 of

cycle

c. oral tranexamic

study D.COCP

AAAAAAAAA

27. Post hysterectomy patient with history of DVTpost operative after


hysterectomy. Now comes with hot flushes. Which HRTis best?

Low dose Oral estradiol

Transdermal estrogen plus

Progesterone Oral

Progesteronealone

Transdermal estrogen

EEEEEEEEEEEEEEEEEEes

28. man with sexual difficulty on resperidone ,what to do?

Ans.aripiprazole

29. 58 years complaining back pain aftergardening o/e tenderness in


lumbarvertebrae L3,L4 & limited flexion & extension, aftergiving
analgesics what is urnext mx?

bedrest

continue

activity

BBBBBBBB

30. Picture of a palm fascia thickening and fibrosis with skin puckering
associated with the ring finger and litter fingerflexion (Dupuytren’s
contracture) . He worked as a farmerfor long time, drinks 4 cans of alcohol
every day and smokes 30 cigs/day. Which of the following is the most
common cause?

a.Farm worker

b.Chronicinfection
c.Vibrating tool damage

d.Smoker

e.Alcoholism

EEEEEEEEEEEEE

31. A lady with BMI of 35 how will u manage herin addition to

exercise,weight reduction and diet?

32. a.Involve in sports

b.Low carb food always

c.Lipase

inhibitor

d.Diuretics

e.Surgery

CCCCCCCCCC

33. Old man 92 years old presented by late some cancer, presented with
acute dyspnoeasomething, tells not to perform any life saving procedure

Arrange meeting with herfamily to

decide Ask hospital forpermission

Referherto palliativecare

Put a sign of DNR on his foot of the bed


CCCCCCCCCCCCCCCCCCCCCCCC

34. 3rd post op woman become agitaed and continuing try to get out of
bed ,vital sign all normal only except O2 is 86% in room air. aftergiven o2
by face mask .whats next?

A blood glucose

level

B alcohol level

C. X ray
CCCCCCCCCCCCCCCCCCCCC

� 40. Tourett scenario- Management

asked.

Resperidone

Planzepine

Mitrazepin

Alprazolam

AAAAAAA

Child with lead level 0.75 ( normal 0.32 given ) . What to do next?�a. Check IQ

b.Educate mother

c. Referforchelation therapy

AAAAAAAAAAAAAAAAAAAAAAAA

41. yearold male with a history of fall on an outstretched hand with pain
and swelling. XRay was given, ( showed scaphoid fracture. Very
clearfracture linethrought the middle)�What is the best treatment option
for this patient�
42. a. Crepebandage�
43. b. Plaster cast

c. Analgesics�
d. Compression screw�
e. Plate fixation�
BBBBBBBBBBBBBBBBBBBBBBBBB

sertraline 100mg controlled depression. now taking esctasy for .1


 .recreational purpose. wants to effect of both
a. worsen depression
 b. leads to psychosis
c. synergistic effect
 cccccc

old woman, post surgery develops painless distention. xray of .2


.huge dilated bowels. initial tx
a. nasogastric aspiration
b. rectal flatus tube
c. colonic compression
aaaaa
https://www.uptodate.com/contents/postoperative-ileus?
search=post%20operative%20paralytic
%20ileus&source=search_result&selectedtitle=1~60&usage_type
=default&display_rank=1#h95702274

peutzjeghers syndrome picture.intussessption recall .3

 ra antibody asked .4


 ans: anti ccp

20year hx of a woman with controlled microscopic polyangitis .5


on cyclophosphamide develops hematuria which is painless. inv
asked
 a. cystoscopy
b. anca
none …………stop cyclophosphamide 1st…………

 .diabetic ulcer. cause of forming ulcer .6


ans: neuropathy

year old woman, choatic anxious agigated, multiple wrist 23 .7


lacerations. this happens post breakup with boyfriend.
admitted.shouts on nurses. found with inflated bp apparatus
:around neck. cause
a. passive agression
b. acting out
c. schizophrenia
 d. genuine attempt of suicide
ddddddddddddddddddddddd

young woman (dont remember age but surely above 20) has all .8
voilent and criminal record. left home at 13 years and doesnt talk
:about past. dx
a. borderline
 b. conduct
 c. no option for antisocial
aaaaaa

daughter brings old father to clinic. he is breathless and admits .9


being breathless. no hx of smoking given. no hemoptysis. x ray
 😞  given which i cannot diagnose
 a. lung cancer
b. lung fibrosis
 c. can’t remember options
mostly bbbbbbbbbbbbbbbb ?....needs whole scenario

iraqi man with persistent dry cough and an xray which i think .10
 .has an apical haziness so i think it was tb

 .pulmonary atelactasis scenario .11


 ans: chest physio
hypertensive retinopathy picture .12

 retinal deattachment picture .13

 .ecg of svt tx asked .14

.chads score. warfarin question .15

a woman with 20 year history of seizures controlled on .16


.sodium valproate now considering pregnancy
 a. cease anticonvulsant
b. continue valproate
 c. decrease dose of valproate
aaaaaaaaaaaaa…………..if controlled for 2 ys
cccccccccccccccc……….if less than 2 ys ( one drug with lowest
possible dose .( best is shift to carbamazepine )

pcos scenario. lh was twice as much as fsh. asked what would .17
 .support dx
 a. ultrasound
 b. progesterone
c. oestrogen level
d. testosterone level
dddddddd

diabetic patient with reduced visual acuity which gets worse .18
with light. picture given of eye but i am not sure if it was cataract
or what. tx asked
a. intraocular lens transplant
b. laser phocoemulsification
c. iridotomy
bbbbbbbbbbbbbb…………cataract

a child sustained an injury to the lower thigh in a football


game. x-ray shows elevation of the periosteum and
callsus formation without involving epiphysis. what is the
?diagnosis in this child
a. osteomyelitis
b. osteosarcoma
c. osteoclasoma
bbbbbbbbbbbbbbbbbbbbbbbbbbbbb

year old boy says he has to count till 20 other wise her 17
mom will be killed, they has a minor accident 3 months ago,
boy is keeping himself into the room most of the time saying
he is hearing the voices but couldn't recognise it, whats the
,diagnosis
,ocd
,sever depression
shezophreniform disorder
ccccccccccccccccccc

years-old polycystic ovarian syndrome patient with 32


previous one child of 3 years age is with infertility for two
 ? years. what is the next appropriate treatment for infertility
a) metformin
b) clomiphene
c) cocp
bbbbbbbb

picture of a palmar fascia thickening and fibrosis with skin


puckering associated with the ring finger and little finger
flexion.he sustained wound on that arm 1 month back but it
was healed. drinks 1 glass of wine every day and smokes
10 cigarette a day. which of the following is the most
?common cause
farm worker
chronic infection
vibrating tool damage
smoker
alcoholism
EEEEEEEEEEEEEEEEEEEEEEEEEEEEE

pic of shingles .lesion is there for 1 week.what is the best .3


management of this patient
a.oral steroids
b.regular oral analgesia
c.acyclovir
d.topical steroids
bbbb

old man 60 years old who is a heavy smoker for 40 years .7


comes to you with a swelling in the tonsillar region,o/e there
 ? is a mass in the tonsillar pouch,what is your diagnosis
 a. nasopharyngeal cancer
b. lipoma
 c. lymphoma
d. laryngeal cancer
e. metastatic spread
CCCCCCCCCCCCCCCCCCCCCCCCCC…………only accepted ………
most common cause of tonsillar mass is 1ry scc

man living alone on multiple drugs insomnia and goes out


for evening walk nurse visit him every 2 weeks whats not in
favour of him, multiple drus, living alone, too late visit of
nurse or going for walk in the evening
mlti drug

a lady pregnant 37 weeks came with 24 hour severe headache , black


dots in front of eyes and her bp is 145/90
what will lead to make an emergent intervention?
a)increase knee reflex/clonus
b)increase bp to 150/90 within 30 min
c)leg edema
AAAAAAAAAAAAAAAAAAAAAAA

a man had colorectal cancer surger plus chemotherapy for


6 months noe he came for follow up everything is normal,
,what will you check
cbc
cea
BBBBBBBBBBBBBBBBBBBBBBBBBBB

–yr old female c/o of tiredness, lethargy for 6 month. history of atorvastatin
for hypercholestrolaemia .physical examination is normal.
lab result
serum thyroxine level- 8( n= 10-14)
tsh level – 2.4 (n= 1.2- 4)
thyroid autoantibodies
thyroid usg
iron study
thyroxine 50 microg
mri brain

EEEEEEEEEEEEEEEEEEEEEEEEEEEEEEE

year old woman, choatic anxious agigated, multiple wrist 23


lacerations. this happens post breakup with boyfriend.
admitted.shouts on nurses. found with inflated bp apparatus
:around neck. cause
a. passive agression
b. acting out
c. schizophrenia
d. genuine attempt of suicide
dddddddddddddddddddddddd

pt had mva he has low bp and low pulse what will indicate
,to go for laproscopy
splenic hematoma
.blood into pertonium
bbbbbb

child present with fever, neck stiffness, anterior frontanellae full ,


lymphocytes 5400cumm(n <5) , glucose is normal ,no organism on gram
stain, meningitis like scenario
intravenous phenytoin
rectal diazepam
intramuscular morphine
oral paracetamol
intravenous dexamethasone
DDDDDDDDDDDDDDDDDDDDDDD…..VIRAL MENINGITIS

Steroids

 Current evidence suggests that steroids may reduce the risk of


hearing loss in bacterial meningitis.
 Steroids are not recommended in neonates due to concern
regarding effects on neurodevelopment.

Management of viral meningitis

 Admission is required if bacterial meningitis cannot be excluded or


intravenous hydration is required.
 Ensure adequate analgesia

https://www.rch.org.au/clinicalguide/guideline_index/Meningiti
/s_Guideline

q184103- a lady came with her daughter to your office .she


said that her 8 year old daughter doesn't play with her
friends and her social relation with others is poor since 2
months ago and she is annoyed from the older students .
mother is divorced from her father years ago and is in a
new relation with a young man since 3 months ago. what is
?your next management
a-refer to child protection service
b-arrange a consultation with psychologist/psychiatrist for
her
c- talk to the mother without girl
d- arrange a meeting with her biological parents

AAAAAAAAAAAAAAAAAAAAAAAAAAAA

an old man with acute onset pain in the lower back with some urinary
symptoms (think it was retention).significant weight loss. anal sphincter
tone was normal. on dre enlarged irregular prostate. dullness on palpation
2 finger breaths above symphysis pubis.slightly elevated temp. what is
your appropriate investigation to lead you to diagnosis?
a. chest x-ray
b. ct spine
c. trus
d. bladder scan
e. psa
(no option for mri )
ccccccccc
patient with large hands and jaw comes for assessment. what test will
determine the underlying problem?
a. cortisol level
b.somatomadin
c levela
c. usg adrenals
d. blood sugar
bbbbbbbbbbbbbbbbb…………acromegaly………best answer is
insulin like growth factor or growth hormone

a patient was sent on a stess leave after employer finds .27


him using cocaine. the patient admits using cocaine and
also has memory impairment, agitation and is also irritable,
what is the next step
a. urine drug screen
b. collateral history from employer
c. history of sexual abuse
d. past history of drug use
bbbbbbbbbbbbb

ecg now comes with af, hypertension and dm. comes with palpitation. bp
140/90, pr 68/min, pulse is irregularly irregular. he’s taking losartan,
metformin and other drugs for his conditions. what would you give him for
his condition???
warfarin
metoprolol
acei
aspirin
aaaaaa

q2) during early pregnancy, a pelvic examination may


reveal that one adnexia is slightly enlarged. this is most
likely due to
.a) a parovarian cyst
.b) fallopian tube hypertrophy
.c) ovarian neoplasm
.d) follicular cyst
.e) corpus luteal cyst
EEEEEEEEEEEEEEEEEEE

q1) which of the following ultrasound feature is most


?suspicious of malignancy in adnexal mass
a) cyst 8 cm in diameter
b) several internal excrescences
c) cystic with two thin septations
d) free fluid in the pod
e) bilateral ovarian cysts
BBBBBBBBBBBBB

lean woman with ten year hx of diabetes type ii with high


glucose (i think bsl was 16) and glucosuria, protein, and
ketonuria. she takes gliclazide for diabetes. what’s the
?cause of these sx
insulin resitance -
 renal failure -
insufficient insulin production -
ccccccccc

patient who is a known diabetic and hypertensive currently on ramipril and


metformin for the past 2yrs. 2 weeks ago she developed an upper
respiratory tract infection and took amoxicillin for 2days ago and some
vitamin c tablets. now developed right sided tongue swelling and bilateral
wheezing on chest auscultation. he cannot speak clearly.cause is the most
likely dug which can cause this presentation
a. amoxicillin
b. ramipril
c. metformin
d. vitamin c tabs
BBBBBBBBBBBBBBBBBBBBBBBBB

1)post op oliguria,catheterized,has generalized abdominal


tenderness,temp 38.5.next?
blood culture
ct abdomen
doppler usg of bladder
s urea cr
aaaaaa
2)post op oliguria, no catheter and no feverà
bladder scan
serum creatnine
ivu
aaaaaaaaaaa

years female complain of malaise,fatigue. examination ? 25


inr 1.6, calcium 1.9, hb 8, mild microcytic hypochromic
?anaemia,what to do first
a)givevit k
b)give calcium i/v
c)blood transfusion
ccccccccccc

pain in the buttocks&posterior thigh&deadly leg pain after


walking 100 m on normal floor &20 m on unequal land well
felt pedal pulsation next
a..doppler legs
.b.ct angio
c.mri lumbosacral
d.x.ray
AAAAAAAAAAAAAAAAAAAA

pre eclampsiascenarium – dx 25 year old woman has a blood pressure of 160/90


mmhg at week 10 of her first pregnancy. she had recurrent urinary infections in
childhood. her urinalysis shows protein but no blood. her blood tests show:
haemoglobin 109 g/l
serum urea 7.5 mmol/l
serum creatinine 125 μmol/l.
what is the most likely cause of her hypertension?
a chronic glomerulonephritis
b chronic pyelonephritis
c essential hypertension
d pre-eclampsia
e renal artery stenosis
bbbbbbbbbbbbb

4 y.o. boy comes with intermittent abdominal pain. usually it lasts for 12 hours,
sometimes he is vomiting during this time then the pain goes away
spontaneously. o/e no abnormalities mentioned , no fever . today he is also
complaining of the left flank pain. what is the diagnosis
a)hebatoblastoma
b)willms
c)neurobalstoma
d)ureteropelvic junction obestruction
e)pyelonephritis
DDDDDDDDDDDDDDDDDDDDDDDD…………pain + vominting in
kids = UTI

yr old asymptomatic woman present with 25 mm 32


swelling in her right lower thyroid lobe which was found
incidentally in ct scan for the whiplash injury. after taking
fnab there was pssamomma bodies and what
?management
a. review with usg next 6 months
b. review with thyroid function tests next 6 months
c. radio iodine scan
d. right lower lobe lumpectomy
e. total lobectomy with isthmectomy
EEEEEEEEEEEEEEEEEEEEEEEEEEEEEEE

rta pt with several(question said several..no.of ribs not


given) front ribs fracture.. in pain which is radiated to back
side with sob..no vitals given...what will you do next..
(sceanario seemed like flail chest or aortic rupture)
echo
cxr
usg
intubation
ecg
bbbbbbbbbbbbb…….1st step always

patient with type 2 diabetes on metformin 500 mg came to routine health


checkup, lab investigation was done showed as follows 
test result 
random blood sugar 5.6 (4.5-6.5) 
cholesterol (within normal) 
hba1c 6.9% ( normal <6.5%) 
urine dipstick trace of protein
what is your most appropriate next step in management? 
a-commence insulin 
b-increase metformin 
c-commence ramipril 
d-add simvastatin 
e-continue same treatment
CCCCCCCCCCCCCCCCCCCCCCCCCCCCC

yr old male on clopidogrel with h/o drug eluting stent 55 


insertion 1 wk earlier came to your hospital for the fracture
?of femur operation. what is your advice to this patient
a) pospone surgery for about 12 month and then do op
b) immediate platelet transfusion and do surgical procedure
c) give ffp and do surgery now
d) discontinue clopidogrel and do surgery
e) do surgery immediately
EEEEEEEEEEEEEEEEEEEEEEEEEEEEEEEEE

an old lady 67 years is admitted for knee replacement


surgery. prior to admission she used to take 1 to 2 standard
drinks of red wine every evening. she was on
carbamazepine, atorvastatin, ramipril and ibuprofen. on
third in the hospital she develops agitation, tremors and
 .restlessness. what is the cause of this
 a. serotonin syndrome
 b. benzodiazepine withdrawal
.c. alcohol withdrawal
d. psychosis
bbbbbbb

mother living in mining town comes with her daughter for.55


normal blood test. her daughter’s
test show lead level 0.72 (‹0.32)
?what will be your next appropriate step
 a.assess iq testing of child
b.educate mother about environmental risks
c chelation therap
bbbbbb

girl 16 yr,recently became sexually active,lives with )3


parents and dont want to let parent know about her sexual
activity.what is next appropriate action
a.give ocp and tell to use condom
b.askpartens identity and age
c.tell her parents
AAAAAAAAAAAAAAAAAAAAAAAAAAAAAAAA

stress incontinence scenario ,,,,with cough or sneezing ....best next


invest,
ct
urodynamic studies
urine analysis
cystoscopy
BBBBBBBBBBBBBBBBBBBBBBBBBBBB

4 case of a patient bitten by a fruit bat, what’s more important?


a hedraimmunisation
b rabies immunisation
c antibiotics
d tetanus immunisation
BBBBBBBBBBBBBBBBBBBBBBBBBBBBBBB

women underwent total hysterectomy and bilateral oophorectomy , her sister has
ovarian cancer . when to give hrt ?
before operation
immediately after operation
only after symptoms of menopause arise
6 weeks post operative
BBBBBBBBBBBBBBBBBBBBBBBBBBBBB

years old man who suffers from rheumatoid arthritis and 50


who has been treated with prednisolone for 3 years
developed peripheral neuropathy of the lower extremities.
this neuropathy is most likely due to
a. arsenic poisoning
b. thiamine deficiency
 c. development of necrotising arteritis
d. ruptured intervertebral disc
e. vitamin b12 deficiency
CCCCCCCCCCCCCCCCCCCCCCCCCCCC

q4) ca125 (select false)


.a) maybe elevated with menstruation
.b) is raised in 50 percent of stage i ovarian cancers
.c) is elevated in pelvic inflammatory disease (pid)
.d) maybe raised by urinary tract infection
.e) is elevated in patients with pancreatic cancer
DDDDDDDDDDDDDDDDDDDDDDDDDDD…..PILOT

a patient’s urine analysis shows blood +, protein trace. in


the next visit he has the same urine analysis, his urine
microscopy and culture shows no growth, rbcs of
 ?glomerular origin and no casts. probable diagnosis
 a. iga nephropathy
 b. nephritic syndrome
 c. transient benign hematuria
 d. chronic kidney failure
e. thin basement membrane nephropathy

EEEEEEEEEEEEEEEEEEEEEEEEEEEEEE

17 yr old aborginals just got free from detention centre after being there for 3yrs...
he has now severe anger bouts, not always happy and always thinks about his
childhood neglect and maltreatment. now doesn't want to go back to detention for
his behavior. what treatment to give him?
a. cbt
b. dialectal therapy
c. antidepressants
d. pschyanalytic therapy
e. anger management program
AAAAAAAAAAAAAAAAAAAAAAAAAA

q5) which of the following tumors is least likely to be


:hormonally active
.a) sertoli-lying cell tumor
.b) granulosa cell tumor
.c) hair cell tumor
.d) fibroma
.e) thecoma

DDDDDDDDDDDDDDDDDDDDDDDDDDDD

a lady comes to you with weakness of lower limbs. she


drags her foot while walking, dorsiflexion and plantar flexion
is weak. reflexes are exaggerated. equivocal plantar
?response. diagnosis asked
a. cervical spine ms
b. motor neuron disease
c. spinal canal stenosis
d. lesion in cerebral cortex

if both lower limbs = c


 if one side = d
 a not accepted as ul not affected
b not accepted as no fasiculation and no affection of ul

person od on morphine prescribed to them. he gets low bp 80/40 and


tachypnea. what to do immediately?
a. gove iv saline
b give nalaxone
c. oxygen
AAAAAAAAAAAAAAAAAAAAA…..morphine toxicity will NOT cause
tachypnea ……morphine toxicity will not cause tachypnea or low bp.....this
patient has some thing else

15a 40-year-old man presents with a 18-month history of heartburn and


atypical chest pain, both unrelated to food. he noted that elevating his
arms makes it easier to swallow. a month before presentation he
developed intermittent dysphagia to both solids and liquids,
regurgitation, and weight loss of 3 kg. which one of the following is the
most likely diagnosis?
1. oesophageal stricture
2. oesophagitis
3.scleroderma
4.achalasia
5. gastro-oesophageal reflux
44444444444444

a 24-year old female is brought to the emergency .35


department following ingestion of “high dose” of
.paracetamol
on further history she tells you that she has ingested ten
tablets of “usual-strength” of paracetamol one and half hour
ago. she denies any symptoms except nausea and two
episodes of vomiting. she does not smoke but drinks
alcohol at weekends. she is alert and oriented. her
temperature is 37.2c,blood pressure is 110/60 mmhg, pulse
.is 90/min and respirations are 18/min
which of the following is the most appropriate next step in
 ?management
 a. keep the patient under observation for 4 hours
 b. do gastric lavage immediately
c. start oral charcoal and do paracetamol blood level after 3
hours ans
d. do urgent dialysis
e. start n-acetyl cysteine without delay

EEEEEEEEEEEEEEEEEEE…………..symptomatic

https://www.rch.org.au/clinicalguide/guideline_index/Paraceta
/mol_poisoning

a man 25 yr old with colon ca. familial h/0 colon


carcinoma.his 2 siblings has h/0 ureteral carcinoma .wats
-his syndrome
a.familial adenomatous polyposis .1
b.hereditary non polyposis colorectal ca (lynch .2
syndrome)
puetzzeghers .3

BBBBBBBBBBBBBBBBBBBBBBBBB

man after an mva presents to you . ct is given with


hemothorax and pneumothorax. he has anterior chest wall
tenderness. which of the following is the cause of pain in
?the young man
a. pneumothorax
b. haemothorax
c .fractured ribs
d. lung contusion

Cccccccccccccccccccccccccccc

male who has sexual relation with another man found to be


hiv
positive. he refused to tell his wife and insisted to have
unprotected
sexual relation with his wife. what will u do as wife is urpt
too
 tell wife
test wife of hiv
inform health authority
inform police
send them for family counselling
CCCCCCCCCCCCCCCCCCCCCCCCCC

/initial mx of ulcerative colitis with active bleeding


a.salfasalazine
b.steroid

AAAAAAAAAAAAAAAAAAAAAAAA

year old with adhd. refusing to take medicines since few 14


- months.agressive. about to be expell from school
 a. respa depot
 b. tell parents to insure adherence
send state appointed nurse .3
tell parents child has right to refuse meds4 .4

AAAAAAAAAAAAAAAAAAAAAAAAAAAAAA

guy just found out he had huntington’s disease 1 month


ago, brought in by police for driving under alcohol
influence , now he insiders on being discharged home as its
: lambing season , what to do
 a. allow discharge and outpatient detoxification
 b. admit and detoxification
c. refer to huntington’s support group # long term
management

BBBBBBBBBBBBBBBBBBBBB

pms in a lady and sever symptoms like decrease libido and


affecting her marriage
a.primrose oil
b.sertraline 100 mg
c. relaxation
d. pyridoxine
e coc
bbbbbbbbbbbb

patient with weakness of the left upper limb, weakness of


interosseous muscles and right plantar response is
equivocal, left is increased; reflexes are normal. what
?investigation will you do to reach diagnosis
mri cervical spine
ach receptor antibodies
emg
ct brain

AAAAAAAAAAAAAAAAAAAAAAAAA

study for vit d difficiency in ur community?


a) cross-sectional study
b) cohort study
c) case study
d) case series
e) rct
AAAAAAAAAAAAAAAAAAAAAAAAA
a lady brings her 18 month old baby saying she has noticed a swelling in
his abdomen while giving him a shower(not mentioned whether mass is
unilateral or crossing the midline)no other history given.what is the
diagnosis?
a.hepatoblastoma
b.neuroblastoma
c.wilmstumour
d.nephroblastoma
CCCCCCCCCCCCCCCCCCC……….most common intraabdominal malignancy
in kids
If says upper quadrant mass then AAAAAAAAAAAAAAAA

what you will screen before introducing infliximab in a patient with crohn
disease?
a. quantiferon-tb gold test
b. fbc
c. lft
d. ecg
e. urine cytology
AAAAAAAAAAAAAAAAAAAAAAAAAA

pt comes with h/o of unilateral temporal side headache for last 4 hours. he
didn’t experience such type of headache before. he is also complaining of
mild neck pain of same side. pain is not subsiding by taking regular pain
killers. pic of the pt is given below. what will be the next step in
management?
ct head
esr
temporal artery biopsy
cxr
visual evoke potential test

IF ptosis in the pic…………..AAAAAAAAAAAAAAAAA

after a night party a girl comes to the er in the moring with a high fever of
40c, agitated, decreased consciousness level, muscle rigidity, tremor.
what will be the best initial step in management?
i/v infusion
cool blanket and ice pack
i/v diazepam
dantrolene sodium
BBBBBBBBBBBBBBBBBBBBBBBBB

a man was prescribed selegiline for parkinson's. he was on


sertraline 100mg per day for depression and was well
controlled on it. he developed abdominal pain and
diarrhea. his bp was high. what is the next appropriate
?step
a. stop selegeline
b. stop sertraline
c. decrease the dose of sertraline
d. decrease the dose of selegeline
e. stop both sertraline and selegiline

$ EEEEEEEEEEEEEEEEEEEE…………..serotonin

woman around 40 yrs old known asthmatic.her regular medications are


inhaled salbutamol and OCPs. presented with sudden chest discomfort
that did nt respond to salbutamol.on auscultation chest is clear no
wheeze.O2 saturation 86%.ECG shows sinus tachycardia.troponin level
normal.what is the next step in mng.
a.repeat troponin level in 6 hrs
b.Do D dimer level
c.CTPA
CCCCCCCCCCCCCCCCCCC…….pulmonary embolism

ECG of svt.old age patient.b.p 90/45.wht is the initial mng


A.adenosine
b.propranolol
c.DC cardioversion
d.defibrillation
CCCCCCCCCCCCCCCCCCCCC
years old man with renal transplant 8months back, presented with Neck 38
stiffness and meningitis sign start 3 days ago, nothing mentioned about
respiratory symptoms, no fever. On chest xray there is well defined round
opacity in “middle right lung” ( chestxray not given, just mentioned
?secenario ). What is the cause
A. Aspergillous
B. TB
C. Pneumocystis Pneumonia
D. Lymphoma
E. Nocardiosis

BBBBBBBBBBBBBBBBBBBBBBBBB

ECG of RBBB, pt was on antihypertensive,diabetes and now had


 ?drowsinees for last 5/6 days,..what to do next
 A) atropine
b)pacemaker
c)cease antihypertensive

Depends on ECG …..IF bradycardia………..AAAAAAAAAAAAAAA

If no bradycardia…………..CCCCCCCCCCCCCCCC

What is the lifetime risk of developing epilepsy in future in child with history
? of febrile convulsion
a) 1%
b) 3%
c) 30%
d) no risk

AAAAAAAAAAAAAAAAAAAAAAAAAAAAAA

Risk of future afebrile convulsions (epilepsy) is increased by family history of


epilepsy, any neurodevelopmental problem, atypical febrile convulsions
(prolonged or focal).

 No risk factors: risk of subsequent epilepsy approx. 1% (similar to


population risk).
 1 risk factor: 2%.
 More than 1 risk factor: 10%.

/https://www.rch.org.au/clinicalguide/guideline_index/Febrile_convulsion
PIC of hand with some red lesions,scenario of a girl with arthralgia and
lesions.what is the long term treartment,
She was ana positive,dsDNA and RA factor positive.anticcp negative
a.cyclosprin
b.cyclophosphamide
c.hydroxychloroquine
d.NSAID
e.methotrexate
CCCCCCCCCCCCCCCCCCCCCCCCCCCCC….SLE
years old man with COPD was brought to your surgery from nursing 68
home by ambulance. On his way to hospital he received O2, 10 L/m by
—mask. He is still unarousable and his ABG most likely

a) Ph 7.29 PaCO2 65 PaO2 85

b) Ph 7.15 PaCO2 50 PaO2 68

c) Ph 7.25 PaCO2 25 PaO2 100

AAAAAAAAAAAAAAAAAAAAAAAAAAA………..acidosis , increased o2
and deceased co2 …..o2 toxicity

A man, DM type 2, hypertensive, k/c of COPD and a smoker for more than
 20 years. Came
with complain of progressive dyspnea. His face was plethoric and neck
 veins was engorged
till the jaw line. (to me it looked like a scenario of superior vena caval
 obstruction). What to
?do
 a) CT chest
 b) ECG
 c) Echo
d) CXR

AAAAAAAAAAAAAAAAAAAAAAA……………….SVC

Pregnant women first trimester come to you want to stop smoking but
can't, she smokes 25 segrat/ day what will you Give?
A. Non nicotine chewing gum" non Nicotine"
B. Nicotine therapeutic therapy
BBBBBBBBBBBBBBBBBBBBBB………….safe in pregnancy

sepsis due to pyelonephritis, high creat, low BP, oliguric and now also K 6,4. No
ecg changes. What do you do?
1 ivfuro
2 Ca gluc
3 resonium
4 insulin glucose infusion
DDDDDDDDDDDDDDDDDDDDDDDDDDDDDD

Italian 70 year old with jaundice, probably Ca. Now the son calls you and tells
you that he doesn’t want his mother to know the diagnosis. What do you do?
1. Tell the mother an alternate Dx
2. Discuss with other family member
3. Ask your colleague to tell the mum
4. Ask you Healy defence company
5. Refuse request
EEEEEEEEEEEEEEEEEEEEEEEEEEEEEEE

The social phobia but happy gardening qs. They ask what is the most important
other thing in history to ask to know what treatment to start or something like that.
school refusal
alcohol abuse
AAAAAAAAAAAAAAAAAAAAAAAA

Old lady depression scale given 6 (normal is 5). Sensitivty-80% and


specificity- 90% for scale. In general population ppv was 50 % and npv
90% cause asked
 Lady living alone .1
 do some other assesments.2
 has mild depression .3
has severe depression .4

DEPENDS ON WHICH SCALE WAS USED.……


An 11-month-old African American boy has a hematocrit of 24% on
a screening laboratory done at his well-child checkup. Further testing
:demonstrates
hemoglobin 7.8 g/dL; hematocrit 22.9%; leukocyte count 12,200/μL
with 39% neutrophils, 6% bands, 55% lymphocytes; hypochromia on
;smear
free erythrocyte protoporphyrin (FEP) 114 μg/dL; lead level 6 μg/dL whole
blood; platelet count 175,000/μL; reticulocyte count 0.2%; sickle-cell
preparation
negative; stool guaiac-negative; and mean corpuscular volume (MCV)
?fL. Which of the following is the most appropriate recommendation 64
a. Blood transfusion
b. Oral ferrous sulfate
c. Intramuscular iron dextran
d. An iron-fortified cereal
e. Calcium

NOT QUESTION OF AMC …..WILL NEVER ..…………………………


START WITH AFRICAN AMERICAN …..this Q copy paste from USMLE

64 yr old man with low back pain.no symptoms of prostate.crushed vertebrae


t10 L1 seen on bone scan with increased uptake.labs were given
ESR increased more than 100
Ca level was normal.what will u do to know underlying cause
Bone marrow exam
PSA
PET SACN
MRI spine
AAAAAAAAAAAAAAAAAAAA…………will be very hard to be metastatic
prostate and no symptoms of prostate

117. Concerning a mother asking for Antibiotics for her son who is feverish - Her
son visited child clinic which was visited by a girl documented as Meningococcal
Meningitis. The girl came to the clinic from Monday- Wednesday. The child
came on Friday. Hospital gave Rifampicin to all contacts of girl as prophylaxis
A. Give her son Rifampicin
B. Ask her to get her son to ER for review
C. Reassure
CCCCCCCCCCCCCCCCCCCCCCCCCC
A 25 yrs old indigenous lady presented with sob and cough since 3
weeks..o/e temp 37.8,pr,rr,bp,saturation all are within normal
limits,mitral stenosis with bilateral basal crepts ..x ray given with
features likeblateral symmetrical mid lobe pulmonary infiltrate especially
?perihilararea..diagnosis
a.pulmonary edema
 b.pulmonary hypertension
c.rheumatic fever
d.peumocystis pneumonia

CCCCCCCCCCCCCCCCCC………..only accepted……fever with $ of


LHF

Cannot be pulmonary edema as RR and saturation are normal

years female complain of malaise,fatigue. Examination ? INR 1.6, 25


?calcium 1.9, Hb 8, mild microcytic hypochromic anaemia,what to do first
a)givevit K
b)give calcium i/v
c)Blood transfusion

CCCCCCCCCCCCCCCCCCCCCCCCCC………no bleeding mentioned

Pt comes with H/O of unilateral temporal side headache for last 4 hours.
He didn’t experience such type of headache before. He is also
complaining of mild neck pain of same side. Pain is not subsiding by
taking regular pain killers. Pic of the pt is given below. What will be the
next step in management?
CT head
ESR
Temporal artery biopsy
CXR
Visual evoke potential test
AAAAAAAAAAAAAAAAAAAAAA…………if ptosis in the pic…..carotid
dissection
years old child, his BMI 20 came with sore throat. All other family 9
?members are overweight. What will you tell his parents
A- He will grow and become normal
B- He should reduce hours of watching TV

C- He should start on weight reduction regimen


D- replace juices with fruits
E does he take adequte salt

BBBBBBBBBBBBBBBBBBBBBBBBBBBBB

#Surgery#Pt with a history of DVT when young..Undergoing gastric plication


surgery. During surgery they will be using electric blankets for calf Muscle
stimulation. Which of the following would you recommend for the patient. 
1. Nothing 
2. Enoxaparin from before surgery to after surgery. 
3. Enoxaparin from after surgery till discharge. 
4. Enoxaparin from after surgery till 10 days
5. Enoxaparin from before surgery till 10 days after surgery.
EEEEEEEEEEEEEEEEEEE

lady brings her kid.doesnt want her kid to get vaccinated though you have
educated her still she insists on not having her baby vaccinated.what to do
A.infrm health authorities
b.regard her wish
c.vaccinate baby against mothers will
d.no option of involving social worker
BBBBBBBBBBBBBBBBBBBBBB

XRAY of a pt went saudan and now c/o cough anorexia sputum typical xray .73
?of tuberculosis given asking first step in management
a. antibiotics oral
b. isoniazide rifampicin ethumbutol and pyrazinamide
c. admission in isolation
d. bronchodilators

CCCCCCCCCCCCCCCC then BBBBBBBBBBBBBBBB

A girl with depressive symptoms and when asked said “what’s the meaning .74 
of life. What’s most important in management
a. liaison with father
b. discuss safety plan with her and parents

BBBBBBBBBBBBBBBBBBBBBBBBB

yr female epistaxis, headache, malaise, dizziness, LABS: Hb-8.5 Ca-1.9 23 .5


↓ INR-1.5 Ferritin
a. Iron
b. IV calcium
c. IV Vit K
d.blood transfusion

CCCCCCCCCCCCCCCCCCCCCCCC

Patient with weakness of the left upper limb, weakness of interosseous


muscles and right plantar response is equivocal, left is increased; reflexes
are normal. What investigation will you do to reach diagnosis?
A)MRI cervical spine
B)Ach receptor antibodies
C)EMG
D)CT brain
AAAAAAAAAAAAAAAAAAAAAAAAAA……….always 1st step

traveller came back from Africa.have fever(not very high)with tender


splenomegaly.no gisymptoms.hb is 8 g/dl (12-15g/gl).malaria test 3 days
before he left was negative.(no history of taking prophylaxix or net using or
any h/o bathing in lake…).what to do next
a.repeat malarial serology
b.schistosomiasis serology
c.Entamebaehistolytica
d.giardiasis
AAAAAAAAAAAAAAAAAAAAAAAAA

 :Assessment
Who should be assessed

Any patient with a fever who has returned from an endemic malaria region
(See CDCfor more information) in the previous 12 months should be
assessed for malaria (even if prophylaxis was taken).
/https://www.rch.org.au/clinicalguide/guideline_index/Malaria

.
traveller came back from Africa.have fever(not very high)with tender
splenomegaly.no gisymptoms.hb is 8 g/dl (12-15g/gl).malaria test 3 days before
he left was negative.(no history of taking prophylaxix or net using or any h/o
bathing in lake…).what to do next
a.repeat malarial serology
b.schistosomiasis serology
c.Entamebaehistolytica
d.giardiasis
AAAAAAAAAAAAAAAAAAAAAAAAAA

old lady with htnd.m on lots of drugs.now came with increased


drowsiness and loss of appetite.
Given lots of labs
Cbc,rfts,tfts,lfts
Ca phosphorus level.
Everything was normal except urea and creatinine it was high
a.iv normal saline
b.hemodialysis
c.ca gluconate
d.resonium
AAAAAAAAAAAAAAAAAAAAAA…………old recall …..urea and creat
were mildy high………dehydration
If very high urea and creat then dialysis as it will be uremic encephalopathy
-Mesenteric ischaemia…most important identifying point
atrial fibrillation
,bloodydiarrhoea,
abdominal pain

AAAAAAAAAAAAAAAAAAAAAA

Sepsis due to pyelonephritis, high creatinine, low BP, oliguric and now also K
6,4. No ecg changes. What do you do? 
1. iv frusemide
2. Ca gluconate 
3.Ca resonium
4.insulin glucose infusion
DDDDDDDDDDDDDDDDDDDDDDDDDDDD

A patient with left sided fasciculation of deltoid , small muscles of hand. All
upper limb reflexes are normal. However he has lower limb weakness and
extensor planter reflex associated with foot drop. Lower limb sensation
normal .what is the best initial investigation required to make a diagnosis
A. CT scan of Brain
b. mri cervical spine
 c. Electromyelography
d. LP

BBBBBBBBBBBBBBBBBBBBBBBBB

Pt post laparotomy on PCA (pt control analgesia) program giving morphine 1gm
over 5 min which stops automatically in 5 minutes control.Now found out pt is
excessive drowsy but responsible to command. What next in mx?
A) increase o2 supply flow via mask (Changed to this)
B)Change morphin dose to 0.5 mgc.
C/Cease pca and give nurse supervised subcutaneous morphin** D)Naloxone
treatment
E) Intubation
DDDDDDDDDDDDDDDDDDD…………if already on o2 then immediately
give him naloxone

A patient with left sided fasciculation of deltoid , small muscles of hand. All
upper limb reflexes are normal. However he has lower limb weakness and
extensor planter reflex associated with foot drop. Lower limb sensation normal
.what is the best initial investigation required to make a diagnosis
A. CT scan of Brain
b. mri cervical spine
c. Electromyelography
d. LP
BBBBBBBBBBBBBBBBBBBBBBBBBBBBBB

38 years old man with renal transplant 8months back, presented with Neck
stiffness and meningitis sign start 3 days ago, nothing mentioned about respiratory
symptoms, no fever. On chest xray there is well defined round opacity in “middle
right lung” ( chestxray not given, just mentioned secenario ). What is the cause?
A. Aspergillous
B. TB
C. Pneumocystis Pneumonia
D. Lymphoma
E. Nocardiosis
BBBBBBBBBBBBBBBBBBBBBBBBBBBBB

years old man with renal transplant 8months back, presented with Neck 38
stiffness and meningitis sign start 3 days ago, nothing mentioned about
respiratory symptoms, no fever. On chest xray there is well defined round
opacity in “middle right lung” ( chestxray not given, just mentioned
?secenario ). What is the cause

A. Aspergillous

B. TB

C. Pneumocystis Pneumonia
D. Lymphoma

E. Nocardiosis

BBBBBBBBBBBBBBBBBBBBBBBBBBBB

Ctg showing deceleration of fetal heart to 60 twice without uterine contraction.


Term patient vitals stable.. Cervix 5 cm dilated. What will you do?
a. CS
b. Fetal scalp lactate
c. Ventouse #obs
…………..best is stop oxytoxin, give fluid and o2 , left lateral position

Sepsis due to pyelonephritis, high creatinine, low BP, oliguric and now also K
6,4. No ecg changes. What do you do?
1. iv frusemide
2. Ca gluconate
3.Ca resonium
4.insulin glucose infusion
ddddddddddddddddddddddddd

girl with signs and symptoms of depression.what in history will lead you to dx
a.difficulty going to sleep
b.difficulty staying awake
c.early morning wake up and cant go back to sleep
CCCCCCCCCCCCCCCCCCCCCCCC

7.old age lady with polymyalgia rhematica.on oral mpain killers like
codeine paracetamol with dose given.one of her friend who had end stage
cancer and was on morphine gave her morphine and she was using it.now
friend has died.now lady has come for prescription of morphine.what will u
do
a.give her prescription
b.refuse her request
c.report her drug seeking behaviour
d.increse the dose of her oral analgesics
CCCCCCCCCCCCCCCCCCCCCCCCCC

A man present with unable to sit as he is pacing whenever sit down. Last
 ?night, he was injected with depot Zuclopenthixoldecanoate. What to give
 Physostigmine .1
 Benztropine .2
 Diazepam.3
Methyl phenidate .4

Ccccccccccccccc

That what iremeber for today #May 2018


- :Statistics
 Prevelnce ....cross section study
 % SD with 95
 most sensitive factor for study : number of patient
 OB /Gyn
 bishop score 2 ... pg induce -
 OCP ( many questions -
 post menopausal ttt with cancer breast -
 post menopausal ttt DVT -
 post menopausal ttt with hysterectomy-
 ovarian cancer screen -
 round ligament strain -
 2ry amenorrhea -
? ocp first investigation testosterone or transvaginal -
 prepatumhge -
 macrosomia in diabetic mother -
 decrese milk ... due to decrease freq -
 down syndrome screening and investigations -
 microsomia with CMV -
 parvovirus in spherocytosis -
 vit b12 def ( intrinsic factor antibodies) -
 copd and bronchiactasis -
 sigmoid volvolus -
 divericulitis ( treatment and investigation ) -
 Lipoma ( picture )
 Mesothelioma first investigation
 Ecg with old ischaemic changes in inferior leads
 Tia
 Patient with uncontilled fits and her refuse ur advice and continue driving
peripheral viual field loss in long cataract patient fied pupil ... exclude -
 glaucoma
 golf ball hit causing vitreous hge -
 occipital headache and collapse ... normal ct ... subarachnoid hge -
 spinal canal stenosis -
 motor neurone disease (UMNL + LMNL )-
 peripheral limb ischaemia -
 warfarin in CHADVASC score over 2-
 anticoagulant in high risk patient post operative for 10 days-
patient with right hypochondrial pain ct is showing normal right side but -
 left renal mass
most common cause for rebal failure post cadaveric transplant ( no fever -
 no tenderness )
 child in2 years (milestone ) -
 GCS less than 8 ... intubation -
 Meconium aspiration 1 st step (bag and oxygenation .. note no suction )
 SLE antibodies most specific (ANA OR ANTI SMITH ? ) -
 patient on thiazide with ankle pain ... urarecrystale -
 Gouty tophi .... increase allopurinol
 Fracture distal head of fibula ( xray )
vesicular rash on the lateral aspect of shoulder forearm and thumb with -
 index finger ????? First investigation
picture of lipoma -
 HOCM -
... Mms less than 24 asking for consent for electrical therapy-
Mother with severe depession for cancer operation one of daughter refuse
 what u will do
 days post op agitated patient 3
Patient asking for stress sickleave and he admitted that he is cocain
 addict
Mother brought her daughter with vaginal spotting ... sexual abuse you will
?? ckevk her first or u will report directly
 Mother in severe depression not fit on medical ttt for electrical therapy
Ankle oedema in anoexia nervosa
 .Bolimia nervosa most risk factor in family history
Patient with ingestion of metal fireign bodies ( he is antisocial ) what is the
psychiatric disease
 Achalisia treatment cardiac myotomy
Fisrt investigation in boerhave syndrome ( 2 question ) with mangment

pt came with loin pain and hematuria what is the most app step ?
a. ct abdomen
b. ivu
c. usg
d. xray
aaaaaaaaaaaaaaaaaaaaaaaa

you receive a call from the brother of an inpatient who is under your care. he says 
thatb his sister has told him that she will commit suicide in the hospital tomorrow.
he also says not to inform his sister that he told you this. what will be your most
?appropriate action
a. ask brother to ask his sister to talk about her plans with you
b. inform police
c. involve lawyer
d. discuss with the patient about what the brother told you(my ans)
e. don’t do anything as the brother has asked to maintain confidentiality

aaaaaaaaaaaaaaaaaaaaaaaa

year-old woman with previous history of intravenous drug abuse, visited your -23 
office for consultation. she is at 20 weeks of gestation and is hepatitis c carrier.
.she is quite concerned whether her baby would be affected at birth or not
what is the most appropriate management plan for her baby to check hepatitis c
?status
a. check hepatitis c antibodies at 12 months of age
b. check hepatitis c antibodies at 18 months of age
c. do neonatal blood pcr for hcv at birth
d. do maternal blood pcr at 34 weeks of gestation
e. reassure her that her baby would not get hep

bbbbbbbbbbbbbbbbbbbbbbbbbbbbbb

a young male travelled to asia and upon arrival he developed a of cough and mild
fever as well as epigastric pain for a duration of 2 weeks, what is the likely cause?
x ray given
a. viral pneumonitis (exact words)
b. pneumonia pneumococcal
c. fungal infection
d. tuberculosis
AAAAAAAAAAAAAAAAAAAA

patient with decreased hb with cyclical menorrhagia. hysteroscopy and d and c


done. both normal. wts next? with reference plz
a. levonorgestral releasing iucd/ mirena
b. progesterone from day 15-25 of cycle
c. oral tranexamic study
d.cocp

aaaaaaaaaaaaaaaaaaaaaaaaaa

the patient comes with morning stiffness of both wrists for 1 to 2 hours and
both wrists pain. (not mention other joint pain). now patient is concern and comes
with this wrist joint pain. her lab results as follow
hb à reduced
mcv à 77 fl (lower than given normal value)
esr à 75 mm/hr (sure for both values of mcv & esr)
a. nsaids
b. prednisolone
c. hydroxychloroquine
d. methotrexate
e. etanercept

ddddddddddddddddddd…..TTT OF CHOICE

recall of man with fever n swollen knee join on aspiration rhomboid crystals with
gram positive cocci
iv ceftrioxne
colchicine
prednisolone
arthroscopic washout/debridement
aaaaaaaaaaaaaaaaaaaaaaaa

7 days old baby presented with jaundice since 4 days of life, his birth was at term,
not complicated and he is breast feeding, serum bilirubin level was 240 (normal
<200) with 120 (sure) direct bilirubin, his liver is 1 cm palpable below costal
margin. what is the most appropriate next step? 
a.thyroid function test
b.abdominal ultrasound for liver 
c.coombs test 
d.stop breast-feeding and follow up 
e.phototherapy
bbbbbbbbbbbbbbbbbbbbbbbbbb

a primigravida lady 32 weeks of gestation smokes 30 cigarettes per day comes


to your clinic with the complaints of mild pedal edema and sob.on examination
?lung is clear, o2 saturation is 90%. what will be your next step of management
a. v/q ratio
b. cxr
c. ecg
d. echo
e. ctpa

?aaaaaaaaaaaaa………looks like pe

50 old man , ( liver disease scenario with gynecomastia , swollen parotid glad and
testes, agitated , confused ) high temperature 39 , ascites moderate . what to do
next ?
a. fbc
b. abdominal paracentesis
c. check ammonia level
d. abdominal usg
bbbbbbbbbbbbbbbbbbbbbbbbbbbbb

?lady on carbimazole for thyroid, now going to be pregnant.what to do


 a) continue carbimazole
b) stop carbimazole
c)decrease the dose

bbbbbbbbbbbbbbbbbbbb………….shift to propyluracil

male came with scrotal swelling.you diagnose this as varicocele. 


how u will find on exam?
a)the mass is around the testis and postivetransillumintaion test 
b)mass is above the testis and negative tranillumination test 
c) the mass is soft and can get above the swelling
bbbbbbbbbbbbbbbbbbbbbbbbbbbbbbb
mass of va
 of worm

more promin
Tran

https://www.rch.org.au/clinicalguide/guideline_index/acute_scrotal_pain_or_swel
ling/

women on multiple drugs one of them was digoxin, she presented with central
abdominal pain and tenderness. her pulse was irregular. apart from a contrast
?abdominal ct what is appropriate next investigation
 a. abdomen usg
 b. serum lipase
 c. serum lactate
 d. digoxin level
e. inr
ccccccccccccccccccccccccc

the boy was bitten by jellyfish (not mention about box or common). he is .24
?painful and shouting. next mx
a. pour vinegar
b. remove tentacles
c. morphine
d. immerse in the hot water
e. give anti-venom

Dddddddddddddddddddddddddd….first step for the pain

 one new question


yr old man presents with three week history of nausea & vomiting! initially 55
clear fluid, later indigested food particles! last 3 months, he suffered from
abdominal pain radiating to back! most likely dx
ca head of pancreas .1
chronic du .2
ca duodenum .3
 ca body of stomach .4
linitisplastica .5

needs whole scenario ???

a lady has history of pid and endometriosis of the uterosacral ligament surgery
done 4 years ago and she had chlamydia infection treated 2 years ago which of the
following will lead to future infertility
a.surgery
b. chlamydia
c.pid
d.endometriosis

?? needs whole scenario

man has all the enzyme increased . ast(sgpt)< alt(sgot) bilirubin, what is the cause
a) hep a .

b) hep b

c) hep

c d)cmv

e)ebv

aaaaa ? …..if acute elevation of liver enzymes

neonate found apnea in sleeping brought by parents to er no hr apnea with no


 bruises his death is pronounced next
 a.mention on his death paper as cot death(sids)
b.inform child protection cervice
 c.refer him to forensic autopsy
 d.call the coroner
e.arrange funeral for neonate instead of parents

DDDDDDDDDDDDDDDDDDDD
old woman, post surgery develops painless distention. xray of huge dilated
.bowels. initial tx
a. nasogastric aspiration
b. rectal flatus tube
c. colonic compression

AAAAAAAAAAAAAAAAAAAAAAAAA

a man moving to australia from ethiopia.. was done tst. no physical symptoms
?..and signs. no weight loss. tst comes as 12mm induration. what is ur next step
a. treat as latent tb
b. do chest x ray
c. do sputum for afb
..d. repeat tst with interferon-gamma releasing assay

BBBBBBBBBBBBB

old patient who drinks about 10-12 standard drinks per day ,underwent some  
surgery.on 3rd post op day he developed agitation ,o2 saturation 88% .what
 ? next
 blood alcohol-1
 blood glucose-2
 ctpa-3
xray chest-4

DDDDDDDDDDDDDDDDD

lady comes with belief with her head full of worm.some alien want to change .
gene of rice and wheat to all world.previously treated with
?respi,olanzapine,quitiapine.partially controlled symptom.now
clozapine
aripiprazole
ziprasidone

AAAAAAAAAAAAAA………resistant case

year old history hystorectomy and oophorectomy for menorrhagia has -2.8 z  
?score for bone marrow. besides giving calcium, what to give
a) vit d
b) calcitrol
c) estrogen
d) estrogen and progesterone
e) bisphosphonate-alendronate

if has hot flushes………..DDDDDDDDDDD

ld patient. had a vehicle accident. upon eye exam noted to have peripheral loss  
of vision on left only and some cataract. patient never noticed until now. what is
?the next appropriate management/invx
a. explain that it’s normal for old age
b. cataract surgery
c. urgent mri
d. check iop

DDDDDDDDDDDDDDDDDD

year old man present with hypertension with asthma and reflux nephropathy. 50 
lab inv were given. there was high urea,high creatinine and proteinuria 900
?mg/day. what is the choice of anti htn
amlodipine.1
losartan.2
perindropil.3
indapamide.4
metoprolol.5

CCCCCCCCCCCCCCCCCCC

a young woman present with severe chest pain aggravated by inspiration, and
relieved on recumbent position. on exam she has a crunchy systolic sound in left
sternal border, she has h/o pulmonary embolism before. what inv to do to reach
dx??
a) ana(lupus pericarditis)
b) ctpa
c) v/q scan
d) anca
e) ct scan of chest
BBBBBBBBBBBBBBBBBBBBBBBBBBBBBBB
a 42 year-old secretary has lost her job because, due to her perfectionism and
excessive attention to trivial details, she could never finish an assignment on time.
she writes innumerable lists of things to be done and follows rules scrupulously.
she consistently annoys friends and family members with her lack of flexibility
remarks. what is the best treatment for her? 
1) psychodynamic psychotherapy 
2) 2 ) olanzapine 
3) 3) venlafaxine 
4) 4) cbt 
5) 5)exposure & response therapy
DDDDDDDDDDDDDDDDDDDDDDDDDDDDD

50 year old man present with hypertension with asthma and reflux nephropathy.
lab inv were given. there was high urea,high creatinine and proteinuria 900
mg/day. what is the choice of anti htn?
1.amlodipine
2.losartan
3.perindropil
4.indapamide
5.metoprolol
CCCCCCCCCCCCCCCCCCCCCCCCCCCCCCC

what is the lifetime risk of developing epilepsy in future in child with history of
febrile convulsion ?
a) 1%
b) 3%
c) 30%
d) no risk
AAAAAAAAAAAAAAAAAAAAAAAAA

a girl had some superficial wrist injuries. she is normal without suicidal
thoughts .her parents separated last 2 years and she stay with her mom.but also
she wants to move with her father to live with him.who you will inform first ?
a. mother
.b.father
c.grandmother
.both parents
AAAAAAAAAAAAAAAAAAAAAAAAAAA

a 15year old girl having difficulty in concentrating. she constantly fight with
father,need support in school because she cant maintain tasks,use multiple drugs,
like amphetamine,marijuana,tellingits difficult for her to sleep,denied any
hallucination or suicidal thought,what will you do?
liase with father
talk with parents and her 
give ssri
cbt
tell school to support her more
BBBBBBBBBBBBBBBBBBBBBBBBBB

32. patient taking ocps presented with weakness of right arm tone n reflexes
normal cannot move arm actively asking cause
a. tia
b. conversion disorder
c. stroke
d. malingering
BBBBBBBBBBBBBBBBBBB,…………normal exam excludes tia and stroke
If has missed An exam can be DDDD

 multiple gall stone with pain in rhc and small bowel oobstruction
a. cholangitis
b. cholelithiasis
c. ca pancrease??
BBBBBBBBBBBBBBBBBBBBBBBBBBBB

rta pt with several(question said several..no.of ribs not given) front ribs fracture..
in pain which is radiated to back side with sob..no vitals given...what will you do
next..(sceanario seemed like flail chest or aortic rupture)
echo
cxr
usg
intubation
ecg
BBBBBBBBBBBBBBBBBBBBBBB

ecg of ventricular tachycardia.patient is hemp dynamically unstable.you gave him


200 j current with no reversion of rhythm.what's next?
continue cpr for 2 minutes
give 200 j more shock
give amiodarone
AAAAAAAAAAAAAAAAAAAAA

after knee replacement, become confused and sat 88%.


what to do?
abg?
ctpa?
AAAAAAAAAAAAAAAAAAAAAAAAA

pre anaesthetic status of a obstructive sleep apnoea pt.


a abg
b sleep study
c fev
d spirometry
BBBBBBBBBBBBBBBBBBB

picture of a man, 6 hrs of severe right sided hareadache nt experienced before, not
relieved by analgesics,neck stiffness on the same side, photo had miosis and
ptosis of right side wat invest ?
a. mri
b. carotid doppler
c. eeg
d. lp
AAAAAAAAAAAAAAAAAAA

#obs
pregnant lady contacted with cmv affected person. her blood results came are cmv
igm+. next important step
1- terminate pregnancy
2- check her past history previous serology
3- repeat serology ( igm and igg) in the next 2 weeks.
4- give antiviral tx
CCCCCCCCCCCCCCCCCCCCCCCC

40yr man, numbness of 4th& 5th fingers, weak thumb& index opposition, loss of
princer grasp, flexion of wrist and fingers are normal. what is the most likely
site ?
a- cervical neck lesion
b- carpal tunnel nerve entrapment
c- brachial plexus
d- nerve entrapment elbow
e- cervical region
CCCCCCCCCCCCCCCCCCCCC

middle aged man with ecg of svt came with palpitations..what is the cause of this
a.dehydration
b. hypokalemia
c.hyponatremia
bbbbbbbbbbbbbb………..the only accepted here ???

#apgar score
a child was born normally weight 3.2 kg. apgar score was 5 5 in 1 minutes and 8
in 5 mintues. he
has delayed development. wof condition is associated with his developmental
delay?
> 1. paternal uncle has intllectual deficit
> 2. father is alcoholic
> 3. mother has 2 cafe lait spots-pigmented birthmarks
> 4. sister has febrile sizures
> 5. paternal grandmother has hypothyroidism
EEEEEEEEEEEEEEEEE

pain in the buttocks&posterior thigh&deadly leg pain after walking 100 m on


normal floor &20 m on unequal land well felt pedal pulsation next
a..doppler legs
b.ct angio.
c.mri lumbosacral
d.x.ray
AAAAAAAAAAAAAAAAAAAA

rta pt with several(question said several..no.of ribs not given) front ribs fracture..
in pain which is radiated to back side with sob..no vitals given...what will you do
next..(sceanario seemed like flail chest or aortic rupture)
echo
cxr
usg
intubation
ecg
BBBBBBBBBBBBBBBBBBBBBBBBB

ecg of ventricular tachycardia.patient is hemp dynamically unstable.you gave him


200 j current with no reversion of rhythm.what's next?
continue cpr for 2 minutes
give 200 j more shock
give amiodarone
AAAAAAAAAAAAAAAAAAAAAAAAA

after knee replacement, become confused and sat 88%.


what to do?
abg?
ctpa?
AAAAAAAAAAAAAAAAAAAA

xray of a pt went saudan and now c/o cough anorexia sputum typical xray of
tuberculosis given asking first step in management?
a.antibiotics oral
b. isoniazide rifampicin ethumbutol and pyrazinamide
c. admission in isolation
d. bronchodilators
CCCCCCCCCCCCCCCCCCCCCCC

incisional hernia scenario…..this guy had midline incision and developed a


swelling on the incision line…no picture given….asking what will help you
diagnose it?
a. ask the patient to stand and cough
b. ask the patient to lie down and cough
c. ask the patient to lie down and raise the head to chest
AAAAAAAAAAAAAAAAAAAAAAAAAAAA

woman was on trifluperazine .she was ok before with some hand rigidity. she
couldn’t find her doctor after trip.now complain of voice in head.tx
a.trifluperazine
b. resp depot
c. quetiapine
d. haloperidol
e. clozapine
CCCCCCCCCCCCCCCCCCCCCCC

 ld patient. had a vehicle accident. upon eye exam noted to have peripheral loss
of vision on left only and some cataract. patient never noticed until now. what is
the next appropriate management/invx?
a. explain that it’s normal for old age
b. cataract surgery
c. urgent mri
d. check iop
DDDDDDDDDDDDDDDDDDDDDDD
a lady has history of pid and endometriosis of the uterosacral ligament surgery
done 4 years ago and she had chlamydia infection treated 2 years ago which of the
following will lead to future infertility
a.surgery
b. chlamydia
c.pid
d.endometriosis

post op incarcerated hernia 1st day aggitation stem fever sob o2 sat 88% xray
given after o2 what initial step.xray had patches! what initial (xray had patches
here)
iv antibiotics
heparin
thrombolysis
droperidol
DDDDDDDDDDDDDDDDDDDDDDDDD

q40 4 yr old girls came with mom complain is bloody vaginal discharge on
penties for 10 days on examination bloody discharge on vulva.(exact words)
a report cpa as sexual abuse
b exam under aneasthesia
c do swab microscopy nd culture d
stop bubble bath #may
AAAAAAAAAAAAAAAAAAAAA

year old man present with hypertension with asthma and reflux 50 .24
nephropathy. lab inv were given. there was high urea,high creatinine and
?proteinuria 900 mg/day. what is the choice of anti htn
amlodipin.1
e 2.losartan
perindropil.3
indapamide.4
metoprolol.5
CCCCCCCCCCCCCCCCCCCCCCCCCCC

woman with breast cancer history. now presents with severe osteoporosis. does
not want children. whats the best advice
.a.mirenab
pop
c.copd.vit
d and calcium
e.alandronate
EEEEEEEEEEEEEEEEEEEE………………..better raloxifen

one new question 55 yr old man presents with three week history of nausea &
vomiting! initially clear fluid, later indigested food particles! last 3 months, he
suffered from abdominal pain radiating to back! most likely dx
ca head of pancreas .1
chronic du .2
ca duodenum .3
ca body of stomach .4
linitisplastica exact .5
needs whole scenario ???

old man, drag left foot, reflexes on left lower limb are increased, planter flexion .
& dorsiflexion are 4/5, weak ankle movements, equivocal plantar reflex, upper
limb and face are normal xn. lesion site
a. cerebral cortex
b. c spine
c. l5,s1 nerve root
d. common perineal nerve
AAAAAAAAAAAAAAAAAAAAAAAA

patient with weakness of the left upper limb, weakness of interosseous muscles
and right plantar response is equivocal, left is increased; reflexes are normal. what
?investigation will you do to reach diagnosis
a)mri cervical spine
b)ach receptor antibodies
c)emg
d)ct brain
AAAAAAAAAAAAAAAAAAAAAAA

female with bilateral white nipple discharge,childrens are big,h/o sterilizations,


..asthmatic. controlled on inhalatedfluticasone.what do u advise
stop fluticasone
treat with bromocriptine
BB
patien from cambodia, features of jaundice and hepatomegaly with fever,
drowsiness… lots of lab given anfd the blood report favouring of viral picture
without thrombocytopenia
a.hepatitisa
b.dengue
c.malaria
d.cmv
If very high liver enzymes………….AAAAAAAAAAAAAAAA
If not………….CCCCCCCCCCCCCCCCCC

qa well earned business man recently got divorce from his wife and who asked for
a bog compensation from him came with ingest all in the bathroom cupboard.
after basic medical management what will be the next
a. mmse
b. alcohol level
c. urine drug assay
d. some other irrelevant
AAAAAAAAAAAAAAAAAAAA

multiple gall stone with pain in rhc and small bowel oobstruction ..102
a. cholangitis
b. cholelithiasis
??c. ca pancrease
BBBBBBBBBBBBBBBBBB

a 17-year-old girl has taken an overdose of 30 paracetamol tablets 3 hours ago.


she is extremely anxious but otherwise asymptomatic. she has no past medical
history. she weighs 50kg. bp 120/70 mmhg, heart rate is 100 bpm, sao2 98% on
?air. which is the single most appropriate next step
a. activated charcoal
b. gastric lavage
c. n-acetylcysteine infusion
d. paracetamol plasma level 4 hours after overdose
e. urgent liver function tests + clotting screen
DDDDDDDDDDDDDDDDDDDDDDD…….NO SYMPTOMS

y/o female has 1y amenorrhea, 2 years ago pap smear was normal 52
but 4 years ago pap smear
was cervical wart, she has had sexual contact after 1 year, during sex, she
had not any discomfort or
?pain. after that she developed 24 hour vaginal bleeding. cause
a. vaginal atrophy
b. cancer cervix
c. cancer endometr
d. relapse of condyloma

CCCCCCCCCCCCCCCCCCCCCCCCC

1. A young male travelled to Asia and upon arrival he developed a


of cough and mild fever as well as epigastric pain for a duration
of 2 weeks, what is the likely cause? X ray given( just mild hilar
infiltrates..non specific)

a. viral pneumonitis (exact words)

b. pneumonia pneumococcal

c. fungal infection

d. tuberculosis

AAAAAAAAAAAAAAAAAAAA

A scenario of an aboriginal woman presents with mitral stenosis, shortness of ).2


breath, low grade fever, malaise for 6 weeks. Bilateral basal crackles .xray finding
typical of bat wing

a) TB

b) LVH
c) Pulmonary HTN

BBBBBBBBBBBBBBBBBBBBBBBBBBBB

mamu question.3

hutchison freckle ..local excision .4

pt with a history of cll,on fludarabine,cyclophosphamide,rituximab.. presents .5


two days back with with fever nd rigors ..mild pneumonia was diagnosed .started
meropenam and azithromycin..now developed rash (maculopapular on abdomen
not widely spread).what investigation

.a. bone marrow examination

b.skin biopsy

c. drug allergy test

d.immunoflorescence

BBBBB THEN DDDD

Woman 25 years old is going to travel to India presented asking advice about .6

vaccination. She has previous 1 dose of polio vaccine , had DPT at birth, 2 and 4

months age , 1 dose of MMR at 13 month , and tetanus booster at 15 year age.
What

?vaccines should she received before travel

A-MMR, diphtheria and tetanus

B-polio, diphtheria and tetanus

C-polio, MMR, diphtheria and tetanus

D-polio, MMR

E-MMR

CCCCCCCCCCCCCCCCCCCCCCC
Baby born normal vaginally ..no complication. . Everything normal with mild .7
meconium passage..after 1 min baby cnt breath cyaonosed heart rate 40. . What
most appropriate next

a.Oxygen by bag and mask

b.Intubation

c.Aspiration of meconium

d.nasopharyngeal suction

e.cxr

AAAAAAAAAAAAAAAAAAAAAAAAAA

renal transplant hx ..mild neck stiffness..round opacity on xray .no xray given .8
just findings described

a.aspergillosis

b.tb

.c.lymphoma

d.pneumonia

e.nocardiosis

BBBBBBBBBBBBBBBBBBBBBBBBBB

pt with bilateral white nipple discharge ... Asthmatic... Controlled on f .9


luticasone

A:Stop fluticasone

C:Treat pituitary adenoma

CCCCCCCCCCCCCCCCCCCCCCCCCCCC

friable mucosa ..sulfasalazine.10

scenario of prostate cancer T3N0M1.11


a.ebrt

b.radical prostatectomy

c.active surveillance

d.androgen deprivation therapy

DDDDDDDDDDDDDDDDDDD

yr old sexually active ,living with parents..dnt want to inform 12.14


parents.asking for contraception

a.give ocp

b. ask identity of sex partner

c.give ocp nd advise condom

d.tell parents

CCCCCCCCCCCCCCCCCCCCCCC

Inguinal hernia in young man complained he noticed a swelling in his groin .13
after lifting weight yesterday, on exam there was 1cm defect in inguinal area
-with fat protruding in the inguinal canal, wat will be your treatment

a. Reassurance

b. Herniorrhaphy

c. Herniorrhaphy with mesh

d. laparoscopic hernia repair

AAAAAAAAAAAAAAAAAAAAAAAAA

Pt comes with H/O of rt sided headache for last 4 hours. He didn’t ..14
experience such type of headache before. He is also complaining of mild neck
pain of same side. Pain is not subsiding by taking regular pain killers. Pic of the pt
?shows miosis. What will be the next step in management
a. ct

bcarotid.doppler

c.mra

AAAAAAAAAAAAAAAAAAAAAAAA

potop oliguria with cathetr fever.15

postop olguria without catheter.16

lots of prostate BOO.17

hirchsprungs.18

diabetic pregnant with mild renal impairment .risk to baby .19

a.macrosomia

.b.IUGr

IUFD

AAAAAAAAAAAAAAAAAAAAA

20.2min synto

min synto 4 .21

fostrer care picky at food imaginary friend .22

a/.anorexia

b.ocd

.c.sexual abuse

schizophreniform psychosis

AAAAAAAAAAAAAAAAAAAAAAAAAAAAA
?pic of peutz jeghars .complain of abd pain ..rislk of developing .23

a.intussecption

.b.inguinal hernia

couldnt remember exactly

AAAAAAAAAAAAAAAAAAAAAAAAAAA

yrs old child with fever ,abd pain ,vomitting,cough rr 30/min heart rate 120 2 .24
something ,cxr findings explained as some opacity on lt loer lung treatment
asked

a.oral amox

oralroxithromycin

iv penicillin

iv flucloxacillin

CCCCCCCCCCCCCCCCCCCCCCCC

old ladtr weakness lethargy .25

tsh normal t4 mildly decreased

inv

a.thyroid auto antibody

b.thyroid ultrasound

c.mri brain

d reeat tft after 3 months

CCCCCCCCCCCCCCCCCCCCCCCCCCCCCC

that dvt question hx of dm ,htn,ihddvt , af, on ramipril.metformin .26


,warfarin..lost consiousness started amiodarone one week back 4cm swelling lt
? thigh reddened tender no fever .whats the reason
a.dvt

b.cellulitis

c.drug interaction

no option of haematoma

CCCCCCCCCCCCCCCCCCC............... if no fever

week diagnostic downs syndrome ..amnio 15 .27

hep c +ve mom ..avoid fetal scalp sampling.28

postop day 3 hx of alcohol intake agitated ,no mention of dyspnea ..SO2 86% .29
what next

a.blood alcohol

blood glucose

chest xray

d.ctpa

CCCCCCCCCCCCCCCCCCCCCC.............better ABG

A man with history of limb claudication on 100 meters relieved by rest, on .30
examinations there was absent left femoral pulse and absent dorsalis pedis
pulse, ABI was done and it was 0.25. What is the most appropriate test leading
?you to the diagnosis

a. Arteriography

b. Ct angiography

c. compression Doppler ultrasound

d. MR angiogram

e. X-ray

BBBBBBBBBBBBBBBBBBBBBBBBBBBBBBB
duputryens contracture pic given farmer scenario had a cut few days 31
back.alcol intake hx.asking cause

a.alcohol

b.tool related

smoking

due to injury

AAAAAAAAAAAAAAAAAAAAAAAAAAAAAA

.. haemochromatosis in brother ..screening.32

huntington in father ,lady carries gene ,wants to test her 10yr old daughter .33

borderline scenario ..dialectal behavioural therapy.34

asthmatic child present with dka ..followup HBA1c .35

on amisulpride gynaecomastia ..which drug to switch .36

a.clozapine

risperidone

quietiapine

aripruiprazole

DDDDDDDDDDDDDDDDDDDDDDDDDDDDDDDDDD

long scenario female with crf oesophageal motility disorder,tscore -2.7 both .37
femur ;spinre calcium normal.treatment asked

alendronate

' risedronate

calcium vitamin d

strontium

DDDDDDDDDDDDDDDDDDDDDDDDDDD
. low back pain 40 yrs no neurological symptom ..pain extending to foot .38

xray spine

mri spine

observe

CCCCCCCCCCCCCCCCCCCCCCCCCCCCC

scenario of lichensclerosus treatment.39

a.0.5% hydrocortisone

estrogen cream

cautery

AAAAAAAAAAAAAAAAAAAAAAAAAAAAAAA

non accidental injury iun child.40

aspirin 100%.41

sorry thats all I can remember ..mostly recalls ..some new questions but were
doable ..sorry cant remember much .will upload more if I remember . time is not
. that issue .. best of luck .keep me in your prayers

Patient taking OCPs presented with weakness of right arm tone n .32
reflexes normal cannot move arm actively asking cause
A. TIA
B. conversion disorder
C. stroke
D. malingering
BBBBBBBBBBBBBBBBBBBBBBBBBBBBBB

Old woman post op of colectomy with fever 40’C and muscle stiffness
for 2 hrs.She is on multiple drugs including atovastatin. Now comes with
fever 39.7’C and weakness. CK 2500(increased)
A.rhabdomyolysis
B.maglinant hyperthermia

BBBBBBBBBBBBBBBBBBBBBBBBBBBBBBBB

A 79yo anorexic male complains of thirst and fatigue. He has symptoms


of frequency, urgency and terminal dribbling. His urea and creatinine
levels are high. His serum calcium is 1.9 and he is anemic. His BP is
?165/95 mmHg. What is the most probable dx
 a. BPH
 b. Prostate carcinoma
c. Chronic pyelonephritis
d. Benign nephrosclerosis

BBBBBBBBBBBBBBBBBBBBBB

A patient presented to the GP for her regular review. She is well and had
no complaints. Urine RE showed presence of nitrites and Leukocytes.
?Others are normal. What will you do next
Give Trimethoprim
Amoxicillin
Ciprofloxacin
Give no treatment
DDDDDDDDDDDDDDDDDDDDDDD

GIT Q.Patient with complain of dyspepsia , in endoscopy show 5cm #


gastric ulcer , biopsy just shows inflammation cells and helicobacter was
?negative, whats next investigation
ppi.1
Repeat biopsy .2
Breath test .3

CCCCCCCCCCCCCCCCCCCCCCCC
Lady of zuclophenthixol depot for psych issues bcz she’s non compliant with
oral meds. Tried olzapine n resperidone previously but non compliant. Now with
 ?ridigidty n tremor what will u do
 A Switch to quietiapine
 B Resperidal consta
 C Resperidone
D Clozapin

BBBBBBBBBBBBBBBBBB

Old man had h/o angina.He had history of 20 packs of cigratte and 4 drinks of
beer and his cholestrol levelnis high. Now comes to u with complaint with
?increase in frequency of angina. What is ur best management for him
A.stop cigratte smoking
B.Exercise ECG tolerance
C.encourage more exercise
D.reduce alcohol drinking
E.encourage less cholesterol food

AAAAAAAAAAAAAAAAAAAA

Old man coming back from shopping with pain in thigh and leg.on examination
there is tenderness in arm and leg. He is on anti hypertensive drugs, anti diabetic
.drugs and atovastatin 40mg
A.arterial duplex doppler
B.withdrawl atovastatin
C.MRI spine
:Depends on whole scenario
If high CK ............BBBBBBBBBBBB
If symptoms acute ischemia.......AAAAAAAAAAA

Indian university student, 2 months dysuria, hematuria, frequency. All urine test .
 ?normal except RBCs and WBC ++ in urine. Urine culture (-). Dx
 a) Cystoscopy
 b) Renal biopsy
c) Urogram
d) Repeat urinalysis
e) Chlamydia PCR

EEEEEEEEEEEEEEEEEEEEEEEE

years old female migrates to Australia after living 6 years. away from husband. 36
In her country she was abducted and raped by the militia. Now investigation
shows her to be syphilis positive but her husband RPR was negative 1 month
?back. What to do to husband
A. treat husband with ceftriaxone
B. do nothing
C. repeat RPR after 3 months
D. do serial RPR for 3 months

AAAAAAAAAAAAAAAAAAAAAAAAAA

girl has dyspareunia, worry about endometriosis because sister has infertility 17
because of
this. Examination: nodule on ligament on vaginal palpationand some other
description, what is
:important to diagnose endometriosis
A.nodule on ligament
B.dyspareunia
C.menorrhagia
D.family history

AAAAAAAAAAAAAAAAAAAAAAAA

There is Q patient has repetitive previous Hx of colicky abdominal pain


description fits with renal stones, he came with SOB and cough. CXR given lots
of haziness bilateral no mass, no cardiomegaly, investigation asked to reach to a
 .Dx
a. Ca level
b. CT chest
c. CT abdomen
d. echo
IF only stones...........CCCCCCCCCCC
If x-ray shows bilateral hilar
lymphadenopathy..........AAAAAAAAAAAA....Sarcoidosis

A lady has melanoma 0.3mm Breslow thickness and surgery done! She has 3 yr
!and 5 yr old child
?She ask you for her children at risk of melanoma! What will you advise her
A. They have absolutely melanoma if CT2—(Long gene name)
 B. Apply sun cream if they go outside
C. Avoid going outside between 10am-2pm
D. Recommend removal of all dysplastic naevi
E. Skin check 6 monthly
Exact options

DDDDDDDDDDDDDDD..........MORE IMP

.
Arab woman who doesn’t speak English with 16. weeks pregnancy, was referred
by the mid-wife for which she suspects her mental condition. The woman seems
irritable, worried(or anxious) about the people around and she’s uncomfortable
when her husband is not around. She is also irritated with her 2 children. What
condition in her history will be present to lead you to diagnose this patient? (looks
like prodromal symptoms)
Panic attacks .1
Paranoid personality disorder .2
Family member with schizophrenia .3
History of trauma .4
Can be b or c depending on the whole recall

Old woman post op of colectomy with fever 40’C and muscle stiffness for 2
hrs.She is on multiple drugs including atovastatin. Now comes with fever 39.7’C
and weakness. CK 2500(increased)
A.rhabdomyolysis
B.maglinant hyperthermia
Bbbbbbbbbbb

The boy was bitten by jellyfish (not mention about box or common). He is .24
?painful and shouting. Next mx
a. Pour vinegar
b. Remove tentacles
c. Morphine
d. Immerse in the hot water
e. Give anti-venom
DDDDDDDDDDDDDDDDDDDDD...........1st step for the pain

A young male travelled to Asia and upon arrival he developed a of cough and
mild fever as well as epigastric pain for a duration of 2 weeks, what is the likely
cause? X ray given( just mild hilar infiltrates..non specific)

a. viral pneumonitis (exact words)


b. pneumonia pneumococcal
c. fungal infection
d. tuberculosis
AAAAAAAAAAAAAAAAAAAAAAAA

A scenario of an aboriginal woman presents with mitral stenosis, shortness of ).2


breath, low grade fever, malaise for 6 weeks. Bilateral basal crackles .xray finding
typical of bat wing
a) TB
b) LVH
c) Pulmonary HTN
BBBBBBBBBBBBBBBBBBBBBBBBB

mamu question.3
hutchison freckle ..local excision.4
pt with a history of cll,on fludarabine,cyclophosphamide,rituximab.. presents .5
two days back with with fever nd rigors ..mild pneumonia was diagnosed .started
meropenam and azithromycin..now developed rash (maculopapular on abdomen
not widely spread).what investigation
.a. bone marrow examination
b.skin biopsy
c. drug allergy test
d.immunoflorescence
Bbbbb

Woman 25 years old is going to travel to India presented asking advice about .6
vaccination. She has previous 1 dose of polio vaccine , had DPT at birth, 2 and 4
months age , 1 dose of MMR at 13 month , and tetanus booster at 15 year age.
What
?vaccines should she received before travel
A-MMR, diphtheria and tetanus
B-polio, diphtheria and tetanus
C-polio, MMR, diphtheria and tetanus
D-polio, MMR
E-MMR
Cccccccccc

Baby born normal vaginally ..no complication. . Everything normal with mild .7
meconium passage..after 1 min baby cnt breath cyaonosed heart rate 40. . What
most appropriate next
a.Oxygen by bag and mask
b.Intubation
c.Aspiration of meconium
d.nasopharyngeal suction
e.cxr
Aaaaaaaaaa
renal transplant hx ..mild neck stiffness..round opacity on xray .no xray given .8
just findings described
a.aspergillosis
b.tb
.c.lymphoma
d.pneumonia
e.nocardiosis
Bbbbbbbbbbb

pt with bilateral white nipple discharge ... Asthmatic... Controlled on f .9


luticasone
A:Stop fluticasone
B:Treat with bromocriptine
C:Treat pituitary adenoma
BBBBBBBBBBBBBBBBBB...AND CCCCCCCCCC SAME MEANING
.....except if he says removw pituitary then BBBBBBBBBBBBBBBBBBBBB

friable mucosa ..sulfasalazine.10


scenario of prostate cancer T3N0M1.11
a.ebrt
b.radical prostatectomy
c.active surveillance
d.androgen deprivation therapy
Ddddddddfddd

yr old sexually active ,living with parents..dnt want to inform 12.14


parents.asking for contraception
a.give ocp
b. ask identity of sex partner
c.give ocp nd advise condom
d.tell parents
Ccccccccccc
Inguinal hernia in young man complained he noticed a swelling in his groin .13
after lifting weight yesterday, on exam there was 1cm defect in inguinal area with
-fat protruding in the inguinal canal, wat will be your treatment
a. Reassurance
b. Herniorrhaphy
c. Herniorrhaphy with mesh
d. laparoscopic hernia repair
Aaaaaaaaaa

Pt comes with H/O of rt sided headache for last 4 hours. He didn’t experience ..14
such type of headache before. He is also complaining of mild neck pain of same
side. Pain is not subsiding by taking regular pain killers. Pic of the pt shows
?miosis. What will be the next step in management
a. ct
bcarotid.doppler
c.mra
Aaaaaaaaaa

potop oliguria with cathetr fever.15


postop olguria without catheter.16
lots of prostate BOO.17
hirchsprungs.18
diabetic pregnant with mild renal impairment .risk to baby.19
a.macrosomia
.b.IUGr
IUFD
Aaaaaaaaaaaaaaaaa
20.2min synto
min synto 4 .21
fostrer care picky at food imaginary friend.22
a/.anorexia
b.ocd
.c.sexual abuse
schizophreniform psychosis

Aaaaaaaaaaaa

?pic of peutz jeghars .complain of abd pain ..rislk of developing.23


a.intussecption
.b.inguinal hernia
Aaaaaaaaaaa

couldnt remember exactly


yrs old child with fever ,abd pain ,vomitting,cough rr 30/min heart rate 120 2 .24
something ,cxr findings explained as some opacity on lt loer lung treatment asked
a.oral amox
B oralroxithromycin
C iv penicillin
D iv flucloxacillin
Ccccccccc

old ladtr weakness lethargy .25


tsh normal t4 mildly decreased
inv
a.thyroid auto antibody
b.thyroid ultrasound
c.mri brain
d reeat tft after 3 months
Cccccccccccccc

that dvt question hx of dm ,htn,ihddvt , af, on ramipril.metformin .26


,warfarin..lost consiousness started amiodarone one week back 4cm swelling lt
? thigh reddened tender no fever .whats the reason
a.dvt
b.cellulitis
c.drug interaction
no option of haematoma
CCCCCCCCCCCCC...........if no fever

week diagnostic downs syndrome ..amnio 15 .27


hep c +ve mom ..avoid fetal scalp sampling.28

postop day 3 hx of alcohol intake agitated ,no mention of dyspnea ..SO2 86% .29
what next
a.blood alcohol
B blood glucose
chest xray
d.ctpa
ccccccccccccccccccc
A man with history of limb claudication on 100 meters relieved by rest, on .30
examinations there was absent left femoral pulse and absent dorsalis pedis pulse,
ABI was done and it was 0.25. What is the most appropriate test leading you to
?the diagnosis
a. Arteriography
b. Ct angiography
c. compression Doppler ultrasound
d. MR angiogram
e. X-ray
BBBBBBBBBBBBBBBBBBBBBBBBBB

duputryens contracture pic given farmer scenario had a cut few days back.alcol 31
intake hx.asking cause
a.alcohol
b.tool related
smoking
due to injury
AAAAAAAAAAAAAAAAAAAAAAAAAA

.. haemochromatosis in brother ..screening.32


huntington in father ,lady carries gene ,wants to test her 10yr old daughter.33
borderline scenario ..dialectal behavioural therapy.34
asthmatic child present with dka ..followup HBA1c.35

on amisulpride gynaecomastia ..which drug to switch.36


a.clozapine
B risperidone
C quietiapine
D aripruiprazole
DDDDDDDDDDDDDDDDDDDDDDDDD

long scenario female with crf oesophageal motility disorder,tscore -2.7 both .37
femur ;spinre calcium normal.treatment asked
alendronate
' risedronate
calcium vitamin d
strontium
DDDDDDDDDDDDDDDDDDDDDDDD
. low back pain 40 yrs no neurological symptom ..pain extending to foot .38
xray spine
mri spine
observe
CCCCCCCCCCCCCCCCCCCCCCCC

scenario of lichensclerosus treatment.39


a.0.5% hydrocortisone
estrogen cream
cautery
AAAAAAAAAAAAAAAAAAAAAAA

non accidental injury iun child.40


aspirin 100%.41

days old baby presented with jaundice since 4 days of life, his birth was at term, 7
not complicated and he is breast feeding, serum bilirubin level was 240 (normal
<200) with 120 (SURE) direct bilirubin, his liver is 1 cm palpable below costal
 ?margin. What is the most appropriate next step
 a.thyroid function test
 b.abdominal ultrasound for liver
 c.coombs test
 d.stop breast-feeding and follow up
e.phototherapy
Bbbbbbbbb

Woman was on trifluperazine .she was ok before with some hand rigidity. She
couldn’t find her doctor after trip.now complain of voice in head.tx
a.Trifluperazine
b. Resp depot
c. Quetiapine
d. Haloperidol
e. Clozapine
CCCCCCCCCCCCCCCCCCC

year old female have menopause at 55 , present with purulent brown-greenish 68


vaginal discharge for 6 day , she is sexually active, what is the cause , previous
pap smear normal
a. Chlamydia
b.endometrial Ca
 c.gonnorea
d.ovarrain ca
BBBBBBBBBBBBBBBBBBBBBBBBBBBBBB

: Depersonalization and derealization occur in


 Depression
 Schizo
Ocd
mania
all of the above
eeeeeeeeeeeeeeeeeeeeee

ALL patient on cyclophosphamide. you prescribe imipenum and azithromycin.


?then he develops maculopapular rash. what test do you perform
A. CMV test
B. Skin allergy test
C. Immunoflurosence of the skin lesion
ccccccccccccccccccccc....SKIN SMEAR OR BIOPSY

 Patient having murmurs on diff areas asking wat is more significant


 A Aortic stenosis
B Aortic regurgitate
C Mitral regirg

Aaaaaaaaaaaaa
man with fever n swollen knee join on aspiration rhomboid crystals with GRAM  
 POSITIVE COCCI
 A IV ceftrioxne
 B Colchicine
 C Prednisolone
D Arthroscopic washout/debridement
AAAAAAAAAAAAAAAAAAAAAAA

Helen Jones is a 19 year-old secretary who presents with a one year history of
painless post-coital bleeding. She takes a tri-sequential contraceptive pill. Clinical
examination is normal except for a degree of cervical erosion. Her Pap smear is
reported as "inflammatory ". The MOST APPROPRIATE
-:management is
a) Repeat smear after treatment with triple-sulpha cream
b) Change OCP to a more oestrogenic balance
c) Reassure, but review in six months
d) Refer for colposcopy
e) Change OCP to a more progestogenic balance
AAAAAAAAAAAAAAAAAAAAAAAAAAAAA

Recalls 16 May 2018 Istanbul

1. Patient scheduled for elective planned lap cholecystectomy, on warfarin


and other medication, what to advise about warfarin ?
A. Replace with lmwh
B. Stop warfarin and give ffp
C. Stop warfarin and add clopidogrel
D. Continue warfarin
E. Add aspirin
AAAAAAAAAAAAAAAAAAA…………..ELECTIVE cases = stop warfarin 5 days before
the operation and give heparin as bridge therapy

2. Old lady in elderly house found by her nurse masturbating, on psychiatric


medication which is not mentioned, alert and not confused, What is your
action?
A. Increase dose of psychiatric medicine
B. Inform the nurse to allow more privacy to the patient
C. Inform the nurse to shift the patient to a 4 patient bedroom
D. Inform the patient about health hazards of her act
BBBBBBBBBBBBBBBBBBBBBBBBBBBBBBB

3. Ecg of svt, young patient collapsed suddenly at school during class,


awakened after 1 minute, taken by ambulance to the ER vitals: bp 140/80;
HR 76, RR 18, ecg done in ER shows narrow complex SVT, what is the
initial management ?
A atropine
B Adenosine
C carotid sinus massage
D Cardioversion
E defibrillation
CCCCCCCCCCCCCCCCCCCCC………… stable SVT ….the heart rate mentioned
in the case absolutely wrong recaller

4. Young patient with lethargy and malaise, FBC shows Hb 10.2 with normal
wbc and platelet count.. whats the best next investigation ?
A vitamin b12
B electrophoresis
C iron study
D FOBS
CCCCCCCCCCCCCCCCCCC……….only accepted……..most common cause of
anemia is iron deficiency anemia
except if scenario of thalassemia………..BBBBBBBBBBB

5. Carotid artery dissection

6. 29 yrs old woman complains of weight loss and lethargy and malaise, she
noticed passing more frequent urine in day about 8 times and 2 times in
the midnight.. Investigations revealed HbA1c 11.9, Fbs 9.8 mmol. What
will be the best initial treatment ?
A metformin
B insulin glargine
C Gliclazide
AAAAAAAAAAAAAAAAAA……………best initial drug …….DM
7. 57 old male patient arrived at the ER with lethargy, fever 38, distended
abdominal wall with shifting dullness (picture given showing dilated veins
and distended abdomen), what investigation will identify the cause of his
condition?
A LFT
B USG
C abdominal CT
D paracentesis and fluid cytology
DDDDDDDDDDDDDDDDDDDDDDDDD

8. 32 old young man came for councelling and checkup about colorectal
carcinoma, he has no previous history, nad on examination, he told you
that his father died at age 40 because of CRC, what will you do ?
A advise no further investigation is required
B order FOBS
C refer for colonoscopy
D order CEA
AAAAAAAAAAAAAAAAAAAAAAAAAAAAAAAAAAA

Category 2

one 1st-degree relative with cancer diagnosed before 55 ys -1

two first-degree relatives with cancer diagnosed at any age-2

one first-degree relative and at least two second-degree relative -3


.diagnosed with colorectal cancer at any age

iFOBT  performed every 2 years from age 40 up to age 50, colonoscopy


.every 5 years from age 50 to age 74

 low-dose (100 mg) aspirin daily

9. Young female schizophrenic patient in psychiatric ward suffers ulcer in


right arm, you where informed by the nurse that the patient will not
accept any intervention for this ulcer. What will be your action ?
A call her parents for consent
B obtain medical board power order to treat the patient
C refer to surgical team
D take a swab
E neglect the ulcer
DDDDDDDDDDDDDDDDDDDDDDDDDDDD

However, the Mental Health Act permits an authorised psychiatrist to


:make a treatment decision for a patient who
 does not have capacity to give informed consent to the treatment
proposed by the authorised psychiatrist or
 has capacity to give informed consent to the treatment proposed by
the authorised psychiatrist but has not given informed consent to that
treatment.

The authorised psychiatrist can make a treatment decision for the patient if
the authorised psychiatrist is satisfied that there is no less restrictive way
for the patient to be treated other than the treatment proposed by the
.authorised psychiatrist
The Mental Health Actdoes not permit an authorised psychiatrist to make
a treatment decision about electroconvulsive treatment or neurosurgery
for mental illness for a patient. See electroconvulsive treatment and
.neurosurgery for mental illness for more information
https://www2.health.vic.gov.au/mental-health/practice-and-service-
quality/mental-health-act-2014-handbook/recovery-and-supported-
decision-making/informed-consent

10.15 years old female came to general practice for councelling about
Huntington disease. Her mother is affected by the same disease. What will
you do ?
A perform genetic councelling for the patient alone
B perform genetic councelling for the girl with her mother
C arrange family meeting for councelling the girl
D call her parents to obtain consent for counseling
OLD RECALL…………DO COUNSELLING FOR THE WHOLE FAMILY .………
11.4-60 yrs old male presented with complaints of right sided headache since
last 6 hrs. He never experienced such headache before. No abnormality
O/E, except as given in pic ( right sided very very mild ptosis with
meiosis ) . Next inv…
A. CT Head
B. Right sided Carotid doppler

C Similar pic

AAAAAAAAAAAAAAAAAAAAAAAAA

12.young girl came to the clinic asking about contraception pills, she Is 19
years with normal examination, she said that she recurrent attacks of
headache that is associated with naisea and photophobia for which she
takes sumtriptan, what is the best for her ?
A mirena
B progesteron inly pulls
C estrogen patches
D combined oral (coc)
BBBBBBBBBBBBBBBBBBBBBBBBBBB…migraine

13.Known asthmatic patient presented to ER with status asthmaticus, before


arrival to hospital he took salbutamol inhaler but didn’t improve his
symptoms, what should be given next ?
A salbutamol iv
B oral corticosteroid
C intubate after sedation
D magnesium sulphate im
BBBBBBBBBBBBBBBBBBBBBBBBBBB

CCCCCCCCCC and DDDDDDDDDDDD…………last lines


14.Small child complains of irritating cough, looks toxic, with difficult
breathing, examination showed clear chest to auscultation, what would be
the causative organism ?
A RSV
B hemophillus influenza
C influenza virus
BBBBBBBBBBBBBBBBBB…………toxic look, very ill = epiglottis

15.2 questions on Australian cardiovascular risk chart ( so lucky !)


16.Question about positive and negative predictive value

17.Scenario of meningitis suspected in a clinic, u advised referral to


hospital .. what should be done till reaching the hospital ?
A ceftriaxoneinj
B blood cutlure
C CT head
D urine analysis
E give analgesic
BBBBBBBBBBBBBBBBBBBBBBBBBBB then AAAAAAAAAAAAAAAA

Key Points

1. IV ceftriaxone / cefotaxime should be given as soon as


meningococcal disease is suspected. If unavailable, give penicillin.
2. If IV access cannot be obtained within 15 minutes, administer IM or
IO ceftriaxone/cefotaxime, or penicillin.
3. No investigation should delay antibiotic administration. Collect blood
culture prior to antibiotics.

1.Melanoma eye pic


Exactly this pic
A. Refers to plastic surgeon
B. Wide excision
C. Chemo therapy
AAAAAAAAAAAAAAAAAAAAAAAAA

2.baby after 6 hours of delivery develops jaundice. Asking Cause


A) Neonatal sepsis

B) Hemolytic anemia

C) Beast feeding

BBBBBBBBBBBBBBBBBBBBBBBBBBB

1.Study on Aspirin effects on preventing MI on 100 people. If those who took


aspirin of the 100 ppl only 1 person had a MI. Of those who didn't take aspirin 2
ppl had MI. What is the increase in relative risk in people not taking aspirin?
a. 1%
b. 10%
c. 100%
d. 200%

CCCCCCCCCCCCCCCCCCCCCCCCCCCCCCCC

3. What study will you do to find association between case of rotavirus


admission in hospital and birth weight in first year of life?
A) case control
B) cohort
C) case report
D ) Systematic review

AAAAAAAAAAAAAAAAAAAAAAAAAAA
4. A pharmaceutical company contact you about a drug which was developed
and has been approved by the council to distribute in the market. The drug
reduces stroke risk in HTN &AF patient with 40 % less GI bleeding. Which of the
following is the most important thing to evaluate?
A)Common side effects
B)Absolute risk in the studied patient
C)Number of people involved in the trial
D)Cost of the drug

AAAAAAAAAAAAAAAAAAAAAAAAAAAA

5. There was a patient who undergone stress ECG for ischemic cardiac disease.
his results came positive.
Positive predictive value 55%
Negative predictive value 90%
Chances of patient having the cardiac disease

A)55
B)90
C)45
D)35

AAAAAAAAAAAAAAAAAAAAAAAAAAAA

6.80yr old man presented with his wife. Wife says he sometimes gets confused
and forgetful. He still drives and has stopped reading newspapers. MMSE was
26/30. CT scan of the brain is normal. What is the most appropriate
management?
a. Cease driving at night
b. Repeat MMSE after 3 months
c. MRI of brain
d. Encourage physical and mental activities
e. exclude treatable causes of dementia

DDDDDDDDDDDDDDDDDDDDDDDDDD
7. There was a guy brought by wife that was low motivated, stop going out with
friends, quit job abruptly, early morning awake.He drink alcohol 2 SD per
day.featuresof depression, asked what help to Dx
a. possessionof firearm
b. suicidal thoughts
c. F Hx of depression

d. weight loss

BBBBBBBBBBBBBBBBBBBBBBBB………….most imp

8. A 15year old girl having difficulty in concentrating. She constantly fights with
father, recently stopped ballet class – ask why “ what’s the point of all this ? “ ,
what is the most important in management?
A)Liase with father
B)Discuss the safety plan with her and her parents
C)Give SSRI
D)CBT
E)Tell school to support her more

Bbbbbbbbbbbbbbbbbbbbbbbbbbbbbb

9. Scenario of woman with melody stuck in her mind. She can't get rid of it and is
causing her to be distressed and making her distracted. What will help you in
diagnosis?
A)Level of insight
B)Mood
C)Suicidal idea

AAAAAAAAAAAAAAAAAAA…………..schizo

10. A woman comes to your clinic. She was prescribed Trifluoperazine for her
condition. She was taking it for 3 years with improvement of her condition. She
says that she discontinued taking her medication for the last 3 weeks because
her doctor was not present for he was in trip, she also said that Trifluoperazine
makes her hand or some muscle part stiff, rigid, and restless. Now she presents
with voices in her head. What is the most appropriate initial choice in
management?
A-Trifluoperazine
B-olanzapine
C-quetiapine
D-Respa depot
E-stop Trifluoperazine

CCCCCCCCCCCCCCCCCCCCCCCCCCC

11. Woman on venlafaxine overdose, Loss of appetite and weight about 10kg last
month, muted and dehydrated anything. After giving medical care and thiamine,
what will you do?
A) Increased venlafaxine dose
B) Change to Mirtazapine
C) ECT
D) Brain stimulation

CCCCCCCCCCCCCCCCCCCCCCCCCCC

12. 12 yr old girl who is taken from her drug abuse parents and now living in
fosters since she’s 2 year old , she likes toys and very proud of her big collection
of toys, plays with her immaginary friend , eat same foods always and very picky,
her carer doesn’t like that , what is the most immediate danger she is in ?
• 1- drug abuse
• 2-schizophreniform disorder
• 3-sexual abuse
• 4-anorexia nervosa
• 5-OCD

DDDDDDDDDDDDDDDDDDDDDDDDDDDD

13. Old lady keep losing track of time, and forgetting things. when you examined
her she became agitated what is in her Mental Status Examination help you in
diagnosis?
1 Constructional Apraxia
2 Orientation
3 Thought

4. Mood

5. Insight

DDDDDDDDDDDDDDDDDDD……………pseudodementia

14. Man 1 week after prostatectomy, his wife came to consultation because she
is concerned her husband is acting different, more irritated, aggressive, shouting.
what in the history will help u reach diagnosis?
A. Worsening symptoms at night
B. marked interpersonal aggression
C. history of cruelty towards animal
D. history of drug use

AAAAAAAAAAAAAAAAAAAAAAAAAAAA

15. A lady presented with pneumonia symptoms. She is oriented and stable. GP
suggests her admission to hospital. She refused and ask for medications. What
will you do?

A) Involuntary admission under Mental Health Tributary

B) Prescribe medications

C) Counsel her why admission is necessary

D) Counsel her family to persuade her for admission

E) Get Colleague’s suggestion for admission

CCCCCCCCCCCCCCCCCCCCCCCCCCCCCC

16. women who has started cleaning her house repeatedly is flirtatious with her
husband s friends and is is showing impulsive behavior for one weak?
a)Hypomania
B)OCD
C)borderline
D)histrionic

AAAAAAAAAAAAAAAAAAAAAAAAAA

17. Young aboriginal male presents to you with insomnia, fear of darkness and
seeing “mamu”. He has been having these symptoms after the death of his
mother. Which of the following should be next step in his treatment?
A-give him benzodiazepine
B-consult to aboriginal health worker
C-urine drug screen
D-antipsychotic
E-drug and alcohol abuse counseling

BBBBBBBBBBBBBBBBBBBBBBBBBBBBBB

18. A pregnant lady came to you with multiple bruises in her thigh as a result of
husband abuse. She is fear for her safety.
What’s the next step in management?
A. Provide an immediate shelter
B. Confront the husband
C. Suggest she call the police
D. Take pictures for reporting
E. Arrange couple meeting

DDDDDDDDDDDDDDDDDDDD then AAAAAAAAAAAAAAAAAAAAA

19. Woman whose son is a drug addict, beats her every day. She is afraid and
presents to you. What will u do for the safety of the woman?
A. Inform her that you are obliged to inform the police
B. Go to the police
C. Tell her to go to refugee shelter
D. Advise her to go support group
E. Send the son to drug addiction service

CCCCCCCCCCCCCCCCCCCCCCC
20. Patient posted on Facebook page about you telling others “not to consult
with this doctor.” What would your response be?
1. Ignore
2. Tell him to come discuss about his post
3. Report the medical defense association
4. Tell him to delete his post

BBBBBBBBBBBBBBBBBBBBBBBBBB

22. A lady present to you and she tells that God tell her `Fast 40 days and time
so that World War end ‘.She is orientated and decline her to test other function.
What is your first line of management?

A) Zuclopenthixol

B) Diazepam

C) Quatiapine

D) Risperidone

E) Clozapine

DDDDDDDDDDDDDDDDDD………..delusion = sntipsychotic…..1st line is respiridone


and olanzapine

23. 40 years old female brought by his friend in emergency,H/O Social phobia for
10yrs..She is alone at home for many years due to social phobia.When at
home,her interest in Gardening,Reading,Now Which History describes her
personality disorder for treatment??
A.School Refusal
B.Family H/O
C. Self-Harm
D. Alcohol abuse

AAAAAAAAAAAAAAAAAAAAAAAAAA

No option for substance abuse


24. Scenario with female patient always angry, and can’t control herself,
unstable relationships. Self-harm was there, treatment was asked
a. anger management
b. dialectal behavioral therapy
c. Psychoanalytical therapy
d. Interpersonal therapy

BBBBBBBBBBBBBBBBBBBBB………….BPD

25. a man works on a farm of another area brought by police for breaking a
window with brick. he said he remember nothing except loss of his job from his
farm. Dx?
1.depersonalization
2.automatism
3.dissociative fuge
4.derealization
5.conversion

CCCCCCCCCCCCCCCCCCCCCCCCCCC

26. Woman after death of her husband due to prostate cancer, lives alone ,6
weeks later presented with loss of appetite, thinks he died because of her
infidelity. She sleeps badly(exact) and had similar episodes after the death of her
child. Prompt treatment asked.
Risperidone
Venlafaxine
ECT
Mirtazapine
Escitalopram

CCCCCCCCCCCCCCCCCCCCCCCC

27. A man was brought by his wife who had complained of sudden onset of
tongue swelling. He is a known diabetic treated with metformin and
chlorpropramide, hypercholesterolemia given simvastatin and hypertension for
which he takes ramipril. His GP prescribed him amoxicillin 2 days ago for a URTI.
On examination, tongue is swollen. He has difficulty speaking and points to his
tongue. Wheezing can be heard on auscultation. Which of the following drugs is
responsible for the patient’s presentation?
A. Metformin
B. Chlorpropramide
C. Simvastatin
D. Amoxicillin
E. Ramipril

EEEEEEEEEEEEEEEEEEEEEEEEEE

28. Pt on Amisupride comes with gynaecomastia. Which drug may be the most
appropriate next drug?

a) aripiprazole
b) Clozapine
c) Olanzapine
d) Quetiapine

AAAAAAAAAAAAAAAAAAAAAAAAAAAAAA

29. Couple come for infertility problem for the last 12 months. On testing
examination and tests of the female are unremarkable. Male has azospermia.
And bilateral absence of vas deferens. Which of the following is most important
test before the starting the treatment of infertility?
A. No testing required as they cannot have a child
B. Testing of both male and female for cystic fibrosis
C. Refer for IVF
D. Serum FSH and LH for male
E. Serum Testosterone level

BBBBBBBBBBBBBBBBBBBBBBBBBBBBBB

30. ECG – HR-140, regular , narrow complex, no ST elevation or depression , P


wave morphology normal

60 yr old man suffered lightheadache and palpitation during jogging in the


morning. He has hypertension for 5 yr and received regular treatment. Now BP
140/90 and vital signs are stable.
His current medication Thiazide and statin. He drinks 2 SD per day. Most likely
cause?

A. Alcoholic Cardiomyopathy

B. Ischaemic heart disease

C. Hypertensive heart disease

D. Dehydration

DDDDDDDDDDDDDDDDDDDDDDDDDD

Exact option

31. ECG – no changes of hyperkalaemia ,HR – around 70,regular rhythm, no


abnormality found. Patient presented after episode of syncope. He has been
taking Ramipril for hypertension. K- 5.5(exact)

What will you do?

A. Cease ramipril

B. Calcium carbonate

C. Insulin & glucose

D. Resonium

E . Pacemaker

AAAAAAAAAAAAAAAAAAAAAA…………5.5 will not cause syncope + normal ECG

Exact option

32. ECG – ST elevation in V1, V2, V3, no arrhythmia detected

Patient has history of chest pain 2 hour ago , symptom relieved and now pain
4/10. He is on atorvastatin and aspirin. On admission, given oxygen. Most
appropriate next step in management?

A. Nitrate

B. Metoprolol

C. Morphine
D. Clopidrogrel

E. Thrombolysis

DDDDDDDDDDDDDDDDDDDDDDDDDDDDD

33. - Heart failure case man with AF on Ramipril, metoprolol, digoxin. Went on
trip for 2 weeks and stopped his medication. Now with bilateral oedema up to
knee in legs.And ankle swellings.Lungs is clear. And the patient is not dyspneic,
What is the most appropriate ( initial )management?

A- Frusemide

B- Recommence all his medications

C- Ramipril

D- Digoxin

AAAAAAAAAAAAAAAAAAAAAAAAAAAAAAA

34.

50 years old patient, 20 years history of smoking. He has 2 year history of


dyspnea and cough with sputum. On examination, dyspnea+, no
orthopnea, not increased JVP, basal crepts+, SaO2- 86%.Chest X-ray was
given –quite similar
Diagnosis asked?
A. Asthma
b. COPD
C. Pulmonary embolism
d. Bronchiectasis
e. Heart failure
EEEEEEEEEEEEEEEEEEEEEEEEEEEEE
Exact options

35.old patient who drinks about 10-12 standard drinks per day ,underwent some
surgery. on 3rd post op day he developed agitation ,O2 saturation 88%.what
next ?

1-blood alcohol

2-blood glucose

3-CTPA

4-Chest X-ray

5-Urine RE

DDDDDDDDDDDDDDDDDDDDDDDDDD

36. Pregnant women 28 th weeks, oral glucose tolerance test 8.9(SURE) what to
do?

a) Refer to dietician

b) start metformin

c) start insulin

d) do hba1c

e) Follow up next week

Exact options

AAAAAAAAAAAAAAAAAAAAAAAAAAAA

37.10 year old girl with known case of Type 1 DM came with mother to GP and
HbA1C is 15%.She tell that glucose monitoring is between 5 to 9mmol/l in last 3
months. What is the cause of this?

A) She has been eating high carbohydrate diets

B) Glucose strips are out of date

C) Repeat HbA1C
D) Her glucose monitoring is incorrect

E) HbA1C is not reliable

AAAAAAAAAAAAAAAAAAAAAAAAAAAAAA

Exact options

38. 18 yr old pregnant women, 30 weeks gestation presented with increasing


dyspnea and orthopnea. On examination, JVP increased, bilateral basal crepts
and mild ankle edema. Most appropriate investigation?

A. ECG

B. CXR

C. Echo

D. V/Q scan

E. Troponin

CCCCCCCCCCCCCCCCC…………HEART FAILURE

Exact options

39. Poorly controlled Diabetic with mild renal impairment asking about risk for
baby?

Exact scenario
A. IUGR
B. Macrosomia
C. Renal agenesis
D. Intrauterine fetal demise
E. low Birth weight

BBBBBBBBBBBBBBBBBBBBBBBB

40. A 44 years old man attends your OPD with complaint of increasing swelling
over last 12 hours on his right thigh. He is a diabetic and hypertensive and has
been previously operated 6 weeks back for a fracture femur. He is on Ramipril,
Simvastatin, Warfarin, Amiodarone, NSAIDs, and antacids. On examination you
find that the right thigh is 4 cm greater in circumference than the left thigh and is
tender. His temperature is 37.8. Which of the following is the reason for his
presentation?
A. Arterial embolus
B. Cellulitis
C. Deep vein thrombosis
D. Edema
E. Drug interaction

CCCCCCCCCCCCCCCCCCCCCCCCCCCCCCCC

41. A known case of DM patient comes with 10 minute of difficulty in speech


and weakness in the right side of the body. On examination , no loss of vision
and no bruit in neck, no cranial nerve involvement, no weakness. Diagnosis?
Exact scenario

Carotid artery stenosis
Vertebra basilar ischemia
Brain stem
Lacunar infarct
Malignering
DDDDDDDDDDDDDDDDDDDDDDDD
Exact options

42.patient with type 2 diabetes on metformin 1.5g/day came to routine health


checkup, lab investigation was done showed as follows , BP 135-85 (sure)
Test result
Random blood sugar 5.6 (4.5-6.5)
Cholesterol (within normal)
Albumin-Creatinine ratio -530 ( normal<300) SURE
Normal value are given
What is your most appropriate next step in management?
a) Ramipril
b) Glipizide
c) Insulin
d) Piloglitazone
e) Sitagliptin
Exact options
AAAAAAAAAAAAAAAAAAAAAAAAAA………proteinuria = ACEI

43. 8 yrs old with type 1 diabetes on insulin glargine. Every morning glucose
levels is high. What to do ?
a. check blood glucose levels at 2:00-3:00 am for 3 days
b. check early morning insulin levels
c. increase the evening intermediate insulin
d. give insulin before breakfast
e. give another dose before sleep
AAAAAAAAAAAAAAAAAAAAAAAAAAAAAAA

45. Young woman presents with epistaxis,fatigue,tiredness for


months,menorrhagia.she is pale ,not in distress,
On investigation Hb-85g/l,MCV-low,Ca-1.9,INR-1.5(normal 1.1)SURE. Which of
the following is appropriate next step ?
A-blood transfusion
B-IV Calcium
C-Oral Iron
D-fresh frozen plasma
E- Vit K injection
Exact options
EEEEEEEEEEEEEEEEEEEEEEEEEEEEE

46. Young man with rectal bleeding and found 10cm friable mass on
sigmoidoscopy. management?

Short scenario given

a.steroid

b. sulphasalazine

c. methotrexate

BBBBBBBBBBBBBBBBBBBBBBBBBBBB

47. Peptic ulcer case with Hpylori positive, took triple therapy
(Amoxil+metro+PPI) for a week or 10 days (not sure). Urea breathe test still
positive after 6 weeks. Asking the reason?
- Resistant to metronidazole

- Resistant to amoxil
- Not reliable urea breath test

- review in next week

AAAAAAAAAAAAAAAAAAAAAAAAAAAAAAA

48. Female patient with RA she takes ibuprofen and methotrexate to control her
disease for 5 years. Recently patient complains of some symptoms (jaundice),
and her labs are given.
protein - 9 (6-8) slight increased SURE
albumin- 5 (3.5-5.5) normal SURE
ALT - >100 (7-56)
AST - >1500 (10-40)
GGT - >100 (0-30)

Liver enzymes are >100 SURE


What is the cause of her symptoms?
a. Methotrexate induced hepatitis
b. ibuprofen induced hepatitis
c. Autoimmune hepatitis
d. Viral hepatitis

AAAAAAAAAAAAAAAAAAAAAAAAAAAAA

49. A mother came with her two children. Mother's brother has
haemochromatosis. What screening test advice would you offer?

a) Screen only mother


b) Screen mother and kids
c) Ask her to come with her husband
d) Screen only the children

CCCCCCCCCCCCCCCCCCCCCCCCCCCCC
50. A case of chronic liver disease presented with ascites and low albumin level,
no flapping tremor. What is the next appropriate management?

a. lactulose
b. albumin infusion
c. fluid restriction
d. spironolactone and frusemide

DDDDDDDDDDDDDDDDDDDDDDDDDDD

51. Child presented with UTI, on USG the right kidney is smaller than the left.
What is the best investigation to assess renal function? Short scenario

a) DMSA

b) DTPA

c) Urine culture

d) CT scan

AAAAAAAAAAAAAAAAAAAAAAAAAAAA

52. 38 years old man with renal transplant 9months (exact) back, presented with
Neck stiffness and meningitis sign start 3 days ago, nothing mentioned about
respiratory symptoms, no fever. On chest xray there is well defined round
opacity in “middle right lung” ( chest X-ray not given, just mentioned secenario).
What is the cause?

A. Aspergillous
B. TB
C. Pneumocystis Pneumonia
D. Lymphoma
E. Nocardiosis

BBBBBBBBBBBBBBBBBBBBBBBBBBBBBBBBBB

Exact options
54. Child with previous URTI,RBS +, Protein + in the urine dipstick test. After two
weeks urine shows only RBC + and from non-glomerular origin. What is the
appropriate investigation?
A. Urine cytology and culture
B. Bladder scan
C. Renal ultrasound
D. Urine culture
E. Serum electrolytes and creatinine

AAAAAAAAAAAAAAAAAAAAAAAAAAAAAA

Exact options

55.45yr old man who is hypertensive and just treated for his helicobacter pylori
with triple therapy. Now comes with urine protein and hematuria with 2 cysts in
right kidney & 3 cysts in left kidney. Diagnosis?

a) PKD
b) IgA Nephropathy
c) Nephrotic syndrome
d) Acute Interstitial Nephritis

e) Phaechromocytoma

AAAAAAAAAAAAAAAAAAAAAAAAAAAAAAAA

Exact options

56. 10yr old child presented with maculopapular rash & recently recovered from
common cold.

Lab inv shows – Hb – 8.6 (SURE)

WBC – 7 (SURE)

Plt – 35 (SURE)

A. Bone marrow aspirate


B. Epstein Barr serology
C. Coagulation study
D. Platelet function test
E. Urine microscopy and culture
DDDDDDDDDDDDDDDDD……………….1st

Exact options

57. Patient with Active RA and chronic pyelonephritis, feels tired and lethargic,
Lab is done and it shows
Hb is round about 9
MCH is low normal
Serum iron decreased
Serum ferritin normal
TIBC decreased
SURE Inv
What is your next appropriate management?
A. Packed RBCs
B. IV iron
C. Low dose corticosteroid
D. Erythropoietin stimulating agent
E. Folic acid
AAAAAAAAAAAAAAAA then DDDDDDDDDDDDDDDDDD

58. A man comes with clumsiness of hands and tripping over. RightUpper limb
weakness and fasciculation.Left Lower limb muscles also has weakness,
increased tendon reflexes. Planter response is equivocal. No wasting. No sensory
loss is given. What is the initial investigation?
A. EMG
B. MRI Spine
C. CT scan
D. CSF examination
E. Cervical spine X-ray
BBBBBBBBBBBBBBBBBBBBBBBBBBBB

Exact options
59. 58yrs old man, drag left foot, reflexes on left lower limb are increased,
planter flexion & dorsiflexion are 4/5, weak ankle movements, equivocal plantar
reflex, upper limb and face are normal exam. Lesion site?
a. Common peroneal nerve
b. Cervical spinal cord
c. L5,S1 nerve root
d. Cerebral cortex
e. Brain Stem
DDDDDDDDDDDDDDDDDDDDDDDDDDDDDDDD

60.
Like this picture – ring enhancement lesion in frontal region
65 yr old man present with three weeks history of headache & vomiting. He has
history of Clark 1 melanoma and surgery 3 years ago. On examination,
temperature 37.3 & no other abnormal findings.
CT given. What is the most likely diagnosis? Exact scenario
a. Brain abscess
b. Melanoma metastasis
c. Glioma
d. Cerebral infarct
e. Tuberculous meningitis
BBBBBBBBBBBBBBBBBBBBBBBBBBBBBBBB
Exact options
61.Patient with parkinsonism like features , history of taking resperidone and
now visual hallucination. His cognitive function is impaired. (Not given
fluctuation). What is the most likely diagnosis?
a. Dementia
b. Lewy body dementia
c. Drug interaction
d. Cerebral infarct
e. Parkinsonism
Exact options
BBBBBBBBBBBBBBBBBBBBBBBBBBBBBB

62.
Nearly like this picture

Man present with 12 hours history of abdominal pain & distension. On


examination, pain around the umbilicus, bowel sounds are increased, rectum
empty. What is the most likely diagnosis?

a. Intussusception

b. Sigmoid volvulus

c. Ca caecum

d. Appendicitis

e. Perforation

Exact options

AAAAAAAAAAAAAAAAAAAAAAAAAAA

63. 74 yrs old male has metastatic prostate cancer, he is taking morphine for
pain relieve and self-prescribed cannabis but recently they are not working and
he is complaining of not improving symptoms, insomnia and agitation ,what will
be better for him?
a. sleep hygiene
b. motivational therapy
c. supportive psychotherapy
d. interpersonal therapy
e. visual photo therapy

Exact options
CCCCCCCCCCCCCCCCCCCCCCCCCCCCCCCC

64. Gout scenario, now presents with acute symptoms,

Lab Urate level little increase and creatinine is ( 1.8 ) (Normal 0.6 – 1.2)) not
mention renal failure just give creatinine value,

what to give? 
a) Prednisolone
b) codeine 
c) naproxen
d) allopurinol 
e) colchicine

Exact options

AAAAAAAAAAAAAAAAAAAAAAAA

65.

35 year old lady come with rash and color change in winter of hands. This is
previously improved by nifedipine.Lab shows

ANA 1/640

Anti dsDNA (+)

Anti CCP (-)

RF (+)

SURE inv

What is the most appropriate treatment?


a. Prednisolone

b. Hydroxychloroquine

c. Methotrexate

d. Naproxen

e. Azathioprine

Exact options

BBBBBBBBBBBBBBBBBBBBBBBBBBBBBBB

66. Similar scenario but not given picture, same options

67. A patient who is going to do surgery for varicose veins in next 3 weeks. He
has been taking clopidrogrel for cardiac stenting for 4 months. What is the most
appropriate management?

a. Reassess need for surgery

b. Do surgery now

c. Change clopidrogrel to LMWH before surgery

d. Stop clopidrogrel and do surgery

e. Change clopidrogrel to warfarin before surgery

Exact options

AAAAAAAAAAAAAAAAAAAAAAAAAAAA

68. Middle age man present with retrosternal chest pain. This is preceded by
vomiting. On examination, reduced breath sounds in lower lobes of lungs and
dullness over these. Next appropriate investigation?

a. CXR

b. Gastrograffinswallow

c. CTPA

d. Non contrast CT chest


e. ECG

Exact options

BBBBBBBBBBBBBBBBBBBBBBBBBBBBBBB

69. A lady with BMI of 35. How will you manage her in addition to exercise for
long term management?
A. Low Carbohydrate food
B. Lipase inhibitor
C. Diuretics
D. Surgery 
E. 4000 kJ/ day= 1000 kcal

Exact options

BBBBBBBBBBBBBBBBBBBBBBBBBBBBB

70. 65 year old man presented with loin pain, urine examination showed
hematuria. What is the most appropriate next step? Exact age

A-X-ray

B-abdominal ultrasound

C-CT abdomen

D-MRI

E-urine culture

Exact options

CCCCCCCCCCCCCCCCCCCCCCCCCC

71. A women on multiple drugs one of them was digoxin, she presented with
central abdominal pain and tenderness. Her pulse was irregular. Apart from a
contrast abdominal ct what is appropriate next investigation? 
A. Abdomen USG 
B. Serum Lipase
C. Serum lactate
D. Digoxin level 
E. INR

Exact options

CCCCCCCCCCCCCCCCCCCCCCCCCCCCCCC

72. A patient with a painless neck swelling moves with swallowing. Now
hoarseness and difficulty in breathing at night.increased over 3 weeks. O/E
smooth swelling in the anterior triangle of neck.dullness of percussion at upper
sternal border. most likely dx? Nearly like this scenario

a. Anaplastic cancer
b. papillary carcinoma
c. MN Goiter
d. hemorrhage in nodule of MND
e. follicular CA

Exact options

BBBBBBBBBBBBBBBBBBBBBBBBBBBBBBBB

73.

Nearly like this picture

Patient is in severe pain and cannot breathe properly. What is the cause of pain
(Exact words)? (A long scenario)
A. Fractured rib
B. Haemothorax
C. Pneumothorax
D. Haemopneumothorax
E. Tension pneumothorax
Exact options
AAAAAAAAAAAAAAAAAAAAAAAAAAAAAAAA

74. 55 year old man, history of appendicectomy and cholecystectomy presented


with 3 wk history of abdominal pain, distension with tympanic bowel. No
vomiting only mild distension. X-ray shows dilated ascending colon, transverse
colon and descending colon, no air. Rectum is empty and normal looking rectum.
No picture X-ray
a. Sigmoid volvulus 
b. Adhesive IO 
c. CA Sigmoid colon

d. Cecal Volvulus

e. Intussusception

CCCCCCCCCCCCCCCCCCCCCCCCCCCCCCCCC

75.

Like this picture but apple core in middle part of picture


65 year old man, History of appendicetomy& cholecystectomy presented with 3
wk history of abd pain, BS exaggerated no air in rectum likely diagnosis?
a. Sigmoid volvulus
b. Adhesive IO
c. CA Sigmoid
d. Pseudo obstruction
e. Intussusception
CCCCCCCCCCCCCCCCCCCCCCCCCCCCCCCCCCC

76. A young man complaint, he noticed a swelling in his groin after lifting


weight yesterday, on examination there was 1cm defect in inguinal area with fat
protruding in the inguinal canal. On ultrasound, the defected is protruded from
deep inguinal ring. What will be the next appropriate treatment?
A. Reassurance
B. Herniorrhapy
C. Hernioplasty
D. Herniorrhapy with mesh repair
E. Laparoscopic mesh repair
Nearly exact scenario
AAAAAAAAAAAAAAAAAAAAAAAAAAAAAAA

77. 56 year old man which extends from umbilicus to xiphisternum. He looked
morbidly obese and had a waist circumference of 110 cm Asking for appropriate
management for the patient. 
A. Weight loss therapy
B. Abdominal binder
C. Herniorraphy
D. Mesh Hernioplasty
E. Observation and Reassurance
Exact options
If hernia…………………DDDDDDDDDDDDDDDD
If not………AAAAAAAAAAAA

79.A man presents with c/o left leg pain, can walk up to 100 meters , due to
pain has to rest for sometimes relieved by rest, on examinations there was , right
leg good peripheral pulses, left leg weak pulses , ABI done and it was 0.25(exact
value). What is the most appropriate test leading you to the diagnosis?
A. Arteriography
B. CT angiography
C. Compression Doppler ultrasound
D. MRI
E. X-ray
Exact options
Bbbbbbbbbbbbbbbbbbbbbbbbbbbbbbbbb

80. A lady had melanoma 0.3mm Breslow thickness and surgery done. She has 3
yr and 5 yr old children. She ask you for her children at risk of melanoma. What is
the most appropriate advice you give her?
A. They absolutely get melanoma if there is CT2----- ( Long gene name)
B. Apply sun cream when they go outside
C. Avoid going outside between 10am to 2pm
D. Recommend removal of dysplastic naevi
E. Skin check 6 monthly
Exact options
D.DDDDDDDDDDDDDDD……….most imp risk factor
MAY 16, 2018 RECALLS 🇵🇵🇵🇵

Pregnant with family hx of DM came in with Normal ogtt results, wat to do?
- no furtherinvestigation
- ogtt at 28 weeks
- repeat ogct at 28 weeks
- BBBBBBBBBBBBBBBBBBBBBBBBBB

Previously obese px gets agitated and cries weighing at scale, bmi 23, takes
lactulose regularly to maintain bm
- body dysmorphicdo
- anorexia
- ocd
- AAAAAAAAAAAAAAAAAAAAAA,,,,,,,,,,NORMAL bmi

Old px underwent surgery of femurfracture, 7 days afterhad dyspneaand


confusion
- PE
- fat embolism
AAAAAAAA

Old woman right sided weakness, confusion, ataxia, BP 190/110, wat next?
- ct scan head
- MRI

- ???BBBBBBBB

Indigent woman with (+)ptb with 3 yearold son (+) quantiferon gold, wat to do
to son?

- repeat quantiferon

- do PPD 10mm

- start isoniazid

- CCCCCCCCCCCCCCCCCCCC

Picof Episcleritis, what to do?recall

-Steroid

-timolol
AAAAAAAAAAAAAAAAAAAAA

Xray with dilated small bowel - sbo

Picof fundoscopy?diagnosis?recall

-Crao

Child with nephroticsyndrome,

proteinuria+++ Wat next?recall


- kidney biopsy

- serum biochemistry

- BBBBBBBBBBBBBBBBBBBBBBBBBBB

Hypogonadotrophins
- pituitary adenoma?

Pregnant with 1L vaginal bleeding


-Absent Fetal heart sounds

14 yr old with fever and hip pain, diagnosis?

-Scfe

-septic arthritis
BBBBBBBBBBBBBBBBBBBBBBBB

Clinical pic of PID, what to do next?


-Cervical swab
-blood culture
AAAAAAAAAAAAAAAAAAAAAA

Man with leg pain, absent pulses, cause?


-Arterial insuff
-venous insufficiency
AAAAAAAAAAAAAAAAAAAAAA

Pregnant dx with genital herpes, mgt?


-Topical acyclovir
-oral acyclovir
-IV acyclovir
BBBBBBBBBBBBBBBBBBBBBBBBBBB

Mechanical back pain recall

Obese man with persistent acid reflux, taken ranitidine and otherantacids, wats
next?
-ppi
-endoscopy
-ureabreath test
CAN BE ANY ………..needs whole recall

Foot drop with intact foot inversion


-Sciatic nerve
-common peroneal nerve
bbbbbbbbbbbbbbbbbbbbbbbb

Bloody nipple dischargerecall - intraductal papilloma

Woman underwent ecg and incident finding of (breast pic- peau d orange,
nipple inversion)
- mastitis
- breast abscess
- ductal carcinoma in situ
- CCCCCCCCCCCCCCCCCCCCCCCCCC

MVA patient, dyspnea, normal chest pe, given cxr

- right pneumothorax

- left pneumothorax

- ruptured hemidiaphragm

- ???

Right sided headache extending to neck, dx?


-Carotid dissection
Child with adhd, treatment?recall
-Methylphenidate
Copd xray
Bradycardia scenario, wat to do?
- stop Verapamil

16 yr old daughterwith family hx of Huntington, parents dont agree with


testing, wat to do?recall
-genetic test at 18 y/o
Hereditary sphero percent recall - 50%
Schizo percent of inheritance from grandmom
Surgeon shouting at staff recall...
-rport to clinical training
Woman depressed on medications, found staring at ceiling recall - catatonia
Sural nerve biopsy recall
Concussion injury, low Na- Siadh
Adrenal failure in meningitis recall
Px with pruritus and jaundice, dx?

Px with progressing cough and dysnea, wat cause?


-amiodarone
-perindopril
-Rosiglitazone
AAAAAAAAAAAAAAAAAAAAAAAA

Man with chf and vent arrythmja, maintained frusemideand amiodarone,


Rashes, appears daytime and exposure to light -Pseudoporphyria

Femoral herniacase- surgical exploration

Pregnant px with abdominal pain recall


-Round ligament pain
-red degeneration of fibroid
AAAAAAAAAAAAAAAAAAAAAAAAA

Secondary amenorrhea
B hcg pregnancy
ACS scenario - Coronary angioplasty
Child with Adhd noncompliant with meds recall - Respadepot
Post partum px with bright red vaginal bleeding recall
- Endometritis

Child with recurrent upperand lower RTinfection, given CBC microcytichypo


pic, wat to investigate?
- sweat test
- serum ferritin

- AAAAAAAAAAAAAAAAAAAAA

Psych ward with px masturbating recall


- respect privacy
Px with alzheimers urinating everywhere
- worsening alzheimers?
-
- Man with laceration, hx of tetanus vaccination 5 weeks ago, no otherhx of
immunization, wat to give
- tetanus vaccine
- ig
- Dpt and ig

CCCCCCCCCCCCCCCCCCCCCC

That what i remeberfortoday #May 2018 Statistics: -


Prevelnce cross section study

SD with 95 %

most sensitive factorforstudy :

numberof patient OB /Gyn

-..............bishop score 2 pg induce

- OCP ( many questions

- post menopausal ttt with cancerbreast

- post menopausal ttt DVT

-post menopausal ttt with hysterectomy

- ovarian cancer screen

- round ligament strain

- 2ry amenorrhea

- ocp first investigation testosterone or transvaginal ?

- prepatum hge
- macrosomiain diabeticmother

- decrese milk due to decrease freq

- down syndrome screening and investigations

- microsomiawith CMV

- parvovirus in spherocytosis

- vit b12 def ( intrinsicfactorantibodies)

- copd and bronchiactasis

- sigmoid volvolus

- divericulitis ( treatment and

investigation ) Lipoma( picture )

Mesotheliomafirst investigation
Ecg with old ischaemicchanges in inferior leads Tia
Patient with uncontilled fits and herrefuse uradvice and continue driving

- peripheral viualfield loss in long cataract patient fied pupil ... excludeglaucoma

- golf ball hit causing vitreous hge

- occipital headache and collapse ... normal ct ... subarachnoid hge

- spinal canal stenosis

-motorneurone disease(UMNL+ LMNL )

- peripheral limb ischaemia

-warfarin in CHADVASC score over 2

-anticoagulant in high risk patient post operativefor 10 days

- patient with right hypochondrialpain ct is showing normal right side but left
renal mass

- most common cause forrebal failure post cadaverictransplant ( no feverno


tenderness )

- child in2 years (milestone)

- GCS less than 8 ... intubation

Meconium aspiration 1 st step (bag and oxygenation .. note no suction )

- SLE antibodies most specific(ANAORANTI SMITH ? )

- patient on thiazide with ankle pain ... urare crystale

Gouty tophi increase allopurinol

Fracture distal head of fibula( xray )

- vesicularrash on the lateral aspect of shoulderforearm and thumb with


index finger????? First investigation
- picture of lipoma

- HOCM

-Mms less than 24 asking for consent forelectrical therapy ...

Mother with severe depession forcanceroperation one of daughterrefuse

what u will do 3 days post op agitated patient

Patient asking forstress sickleaveand he admitted that he is cocain addict


Mother brought herdaughterwith vaginal spotting ... sexual abuseyou will ckevk
herfirst oru will report directly ??
Mother in severe depression not fit on medical ttt for

electrical therapy Ankle oedemain anoexianervosa

Bolimianervosamost risk factorin family history.

Patient with ingestion of metal fireign bodies ( he is antisocial ) what is the

psychiatricdisease Achalisia treatment cardiac myotomy


Fisrt investigation in boerhavesyndrome( 2 question ) with mangment

Initial upright chest xray

 Mother brings her 3 months old breast feed baby,complaining of


baby cries whole night instead of breastfeeding him at night.what
to do now??
A. Give him solid food
B. Give him breast milk whole night
C. Controlled crying behavior
AAAAAAAAAAAAAAAAAAAAAA

 Woman came with complaining of excessive bleeding during her


period.She also lost her weight in last few months, has 2/3 kids
(forgot)telling that her relation with her husband is not going well,not
going for vacation for long time, having some financial crisis ans
renting house.what to do next?
A. Send her to psychiatrist
B. Advice her to go for vacation
C. Send her to gynaecologist
D. Tell her to bring husband next time
DDDDDDDDDDDD only accepted ?

 Adult man has headache,palpitation,sweating high BP 160/100


and a feeling of "something bad about to happen". What is your next
step?
A. Investigate for pheochromocytoma
B. Thyroid function tests
C. Give beta-blockers

AAAAAAAAAAAAAAAAAAAAAAAAAAAA

 PT came with pain on the lateral side of little finger,how to


mx?

A. Rest & elevation


B. Fix with nail
C. K wire insertion
 Mother comes asking you about her 17 year old son because she
saw him wearing his girlfriend's underwear. She is divorced and her
son lives with her. Your advice regarding her son's behaviour:

A. He is a woman trapped in a mans body


B. Its a normal at his age
C. its a consequence of lack of a male role model in his life
D. He is homosexual
AAAAAAAAAAAAAA…………..only accepted


 Regarding hierarchy study,which is the least effective study?
A. case report
B. case control
C. cohort
D. RCT
AAAAAAAAAAAAAAAAAAAAAAAA

 Old pt me with HTN,DM.Long history of drugs …fundoscopy given.


Whats the dx?

A. Diabetic
retinopathy
B. Macular
degeneration
C. Hypertensive
retinopathy grade
3
D.

CCCCCCCCCCCCC

A. Patient recently diagnosed with Huntingtons Disease, he has


been drinking, and acting erratic. The police have brought him
to ED as he was caught drink driving. What is your best next
management?
B. Detoxification in ER
C. To refer him to Huntington support group
D. To send to drug and alcohol clinic
E. Admit and detoxify
EEEEEEEEEEEEEEEEEEEEEEEEEEE

 xray given (joint space is narrow) Football player , can’t stand , painful
ankle joint movement , ask diagnosis ?
A. Fibula fracture
B. Fibula fracture with fracture articular surface of
tibia
C. Fibula fracture with fracture articular surface of
tibia & joint displacement
D. Fibula fracture with joint displacement
E. Fibular fracture with mortise
DDDDDDDDDDDDDDDDDDDDDDD

 Young lady taking antiepileptic for 15 years.Now she


wants to conceive and asking for ur advice
A. Reduce the dose of na valproate
B. Stop the drug and start carbamazepine
C. Continue na valproate
AAAAAAAAAAAAAAAAAAAAA AND STOP IT

Diabetic patient with reduced visual acuity which gets worse with light.
Picture given ,Tx asked

(sorry not getting the exact pic but got very similar to
this)

A. Intraocular lens
B. Laser phocoemulsification
C. Iridotomy
BBBBBBBBBBBBBBBBBBB

 Woman in labour.she has 3 kids and also has h/o previos ceasarean
section. starts bleeding 1000ml,uterus tender, FHR decrese.whats
the dx:
A. Placenta praevia(no options for abruption placenta)
B. amniotic embolism
C. coagulopathy
D. placenta accretea
E. rupture uterus
EEEEEEEEEEEEEEEEEEEEEEEE

woman wants to try billing method for contraception when she can restore
her sexual avctivity,

A. 17 days after ist day of mensturation


B. when vaginal mucous is moist
C. two days of raise of temperature
BBBBBBBBBBBBBBBBBBBBBBBBBB

 Pt with gerd s/s which is controlled by PPI.Endoscopy shows


metaplasia (long scenario with a pic given).what next?

A. Increase ppi dose


B. Do urea breath test
C. Endoscopy after 2 years
CCCCCCCCCCCCCCCCCCC…..esopgagitis

 A student thinks that he has brain tumor. The final exam is near and
he gets more tired , headache and stressed due to tumor. All neuro
exam and CT are normal. After CBT , he still think he has brain
tumor. What is the diagnosis?
A. Hypochondriasis
B. Somatisation disorder
C. Conversion disorder
D. Factitious disorder
AAAAAAAAAAAAAAAAAAAAAAA

1. Anterior MI ecg pic


Man presented with chest pain 1 hour before, pain relief after aspirin.
What to do next?
a. Heparin
b. Streptokinase
c. Angiogram/angiography/plasty?
NEXT AAAAAAAAA
BEST CCCCCCCCCCCCCCCC

2. Maculapapular rash on the leg pic


Boy, came with abdominal pain, joint pain. Rash extended to the
buttocks. What Ix to do?
a.Throat swab
b.Abdominal ultrasound
c.Echo
BBBBBBBBBBBBBBB…………..HSP

3. 33 years old women. Post hysterectomy for menorrhagia. Dexa scan


-2.5 and -1,7. What to give?
a. Estrogen only
b. Alendronate
c. Estrogen progestin
d. Zolendrin acid
BBBBBBBBBBBBBBBBBBBBBBBBBBBB

4. Man , knw case of angina. Came to take his usual prescription from you.
He said his fine just his angina is more frequent now. He is on statin,
acei , aspirin ,and some other drugs. BMI 30, smoker 40packs, total chol
5.he just got divorce and eat outside food most of the time. What to do
next?
a. Increase statin
b. Increase acei
c. Asses motivation to modify risk factor
d. Arrange a ecg stress test
CCCCCCCCCCCCCCCCCCCCCCC

5. Post partum psychosis. Wife is saying terrorist is going to attack.and saying


her husband is a terrorist. Asked what is the most important thing to see for her
immediate managements.
A.Close with baby
B.Breastfeeding frequency
C.Relations with husband
D. Thought content
DDDDDDDDDDDDDDDDDDDDDDDDDDDDD

6. 2.y.o , small painless lump at the anterior triangle. 2cm, firm and non
tender mass. What is next step?
a. FNA
b. Ultrasound
c. Indirect laryngoscopy
BBBB??? NEEDS WHOLE SCENARIO

8. Young boy, back pain and stiffening.xray – bamboo spin-, He is been


taking pcm and pcm/codein for the pain but it is not working anymore. On
exam, lost of lordosis. Tx?
A/
indomethacin
b/ sulfalazine
c/ MTX
d/ physiotherapy

DDDDDDDDDDDDDDDDDDDDDD then AAAAAAAAAAAAAA…..AS

9. Ulcer in medial malleolus . what type of


dressing? a/ hydrogel, bandange and leg
elevation

10.A lady came in abdominal pain. Hx of cholecystectomy and prostetic


valve for which she is on warfarin. INR 5. BP 120/80, HR 80. CT scan
given.
Looks like distented bladded with irregular border

a. Suprapubic chateter
b. Urinary chateter
c. IV VitK
Bbbbbbbbbbbbbbbbbbbbbbbbbbbbbbbb………retention…except if
hematoma clear in CT then CCCCCC

11.Bullemia. Which Family hx is associated?


a. Depression.
b. Obesity.
c. Schizophrenia
BBBBBBBBBBBBBBBBBBBBBBBBBB

12.Man , amputated finger. Hx of change colour during cold. What is the


diagnosis? (Raynoud was not in the answer.)
a. Sclerodrema.
13. 17, y.o girl with primary amenorrhea. Secondary sexual chareteristic at
stage 5. No abdominal pain. What is the next ix?
a/ FSH
b/Pelvis
ultrasound c/
karyotype
CCCCCCCCCCCCC
If no 2ry sexual charaters
do hormonal evaluation

14.xray with foregin body inside. Inmate brought in due to stomach pain.
What is the diagnosis?
a/Factious
Schizophrenia
c/malingering
AAAAAAAAA??.
NEEDS WHOLE

15.a girl, cant remember BMI , exercise too much and keep seeing herself
in the mirror. She said ‘I will die if i put on weight’.
body dysmorphic syndrome
aneroxia nervosa.
If low BMI…….BBBBBBB
If normal BMI……..aaaaaaaaaaa
16.A guy brought his brother because he is very anxious the moment he
step out of his house. If at home, he is well, doing is gardening, reading
book and all the activitiy. Cousin’s wedding is coming up. What TX?
a/
propranolol
b/diazepam
c/SSRI

d/CBt

aaaaaaaaaaaaaaaaaaaaaaa
May 19/5/18 recalls, center- Perth

1. Induction in mild pre eclampsia with bishop score 2

2. Lady presenting with chest pain.O/E her face shows bruises and
lips have cuts. Which system examination reveals diagnosis
a. CVS
b. Endocrine
c. Musculoskeletal
d. Peripheral nerves
CCCCCCCCCCCCCCCCCC……….abuse

3. 15ms old baby crying at night…settles with a bottle of juice…


appropriate advice
a. Give milk
b. Give water
c. Add thickeners to bottle
CCCCCCCCCCCCCCCCCCCCCCCCC

4. CT picture with lady presenting with abdominal pain. Ejection


fraction 20%...how will you manage?
Ct shows head of pancreas ca….ans I think is ercp
5. Waldenstorm macroglobulinemia patient admitted with
pneumonia…amox and ampicillin given…picture shows
widespread rash on chest and abdomen…wht to do next?

6. Rash with pencillin…what alternative drug to give?


a. Cephalosporin
b. Vancomycin
c. Cefazolin
d. Cefotaxime
BBBBBBBBBBBBBBBBBBBBBBBB

7. CTG picture of lady presenting with loss of fetal movements for 2


days…looked like a normal ctg to me
8. Nerve palsy picture with man presenting with itching…discomfort
and diplopia…
a. Left 3rd nerve
b. Right 3rd nerve
c. Left 4th nerve
d. Right 4th nerve
e. 6th nerve
…………CAN BE ANY

9. Scleritis eye picture…management


10.Got 2 leg ulcer picture questions…both below malleolus…one
question was of a patient with past history of DVT and asked
management and the other was with long standing DM on
medication, asking about next appropriate investigation
11.Tourette management
12.ADHD management
13.Schizoaffective disorder
14.Borderline personality disorder
15.School teacher having the urge to recollect everything that
happened that day…..how to manage?
16.Ocd patient asking for non medical management
17.Lady 19weeks gestation…smoker and doesn’t want to quit
smoking…effect on unborn fetus

18.Man presenting with drowsiness since return from Thailand


2months back.loose clothes…unkempt appearance…arousable…
periventricular plaques in brain….what should be checked in the
patient
a.HIV screening
b.ebv screening…rest dn’t rem
AA

Lady wanting to conceive….she has regular 24 day cycle…presenting on


17th day….what hormone to check for her today

a. FSH
b. LH
c. Progestrone
CCCCCCCCCCCCCCCCCCCCCCC
19.Lady comes with complaints of LIF pain more during urination…
she had similar pains 4weeks back which settled on it’s own after
3days…her periods was 9 days back…what is the diagnosis
a. Endometriosis
b. Twisted ovarian cyst
…………..BEST IS OVULATION

20.Lady presenting with fatigue…mild pallor present…FBE shows all


cell indices reduced…what to check in patient?
a. Iron studies
b. Vit b12
AAAAAAAAAAAAAAAAAAAAAAA

21.Lady presenting with forgetfulness….she doesn’t seem to rem


where she has kept her personal belongings…she seems
distressed when gp offered to examine her and declined
examination…what will you find in history?
a. Suspiciousness
b. Hoarding
???

22.TSH 0.01….how to confirm diagnosis…option had generalized


increased uptake in nuclear scan
a. You are a doctor in a town, where 6 people out of 100 are non-smoker. What are
the
chances of stroke. The chances of stroke in smoker is 50%more than non-
smoker. Now

the pharmaceutical company is introducing a medicine which reduces the


chances of

stroke up to 1/3rd in smoker population. What is the percentage of the stroke


population

will get stroke


A. 3%
B. 6%
C. 9%
D. 12%
E. 20%
BBBBBBBBBBBBBBBBBBBBBBBBBB

23.12months old baby brought in by mother. Mother reveals that


boyfriend is abusive towards her but doesn’t want to inform
anyone….what should you do?
a. Call child protection
24.Patient presenting with abdominal pain. Ct pictures shows single
solitary lesion in liver.diagnosis

25.Best screening method should be


a. High sensitivity
b. High specificity
c. High PPV
d. High NPV
AAAAAAAAAAAAAAAAAAAAAAAAAAAAA

26.Hereditary spherocytosis and parvoviral infection


27.Hereditary spherocytosis and inheritance in siblings
28.DKA correction complication in school going child
29.Cause of seizure in meningitis case
30.Old man presents with pain in buttocks and thigh while coming
back from shopping….tenderness +….what will you do next?
a. Duplex USG
b. TFT
31.Lady brought in hospital with some problem….mmse 20….doctor
wants to do interventional surgery and lady agrees to it but
daughter not willing….what to do
a. Since daughter has
power of attorney…go
with her
AAAAAAAAA
32.SVT with bp 80/50…management
33.Boy having urinary protein 3+ and blood 1+ in urine…management

34.Gradual LOV for 5months which improved with pin hole


a. Cataract
b. Diabetic retinopathy
c. Hypertensive retinopathy
d. ARMD
AAAAAAAAAAAAAAAAAAAAAAAAA

35.CO poisoning…management
36.Prolapsed haemorrhoids clinical feature

37.Lady presents with mild temperature and redness of breast


left….non fluctuant area +….what to do after giving antibiotics
a. Breast feed from left breast first
b. Express and discard
c. Aspirate
AAAAAAAAAAAAAAAAAAAAAAAAAAAA

38.Patient controlled analgesia….on fentanyl post op….drowsy but


arousable…pain well controlled…what to do
39.Person with no family history or symptoms wants to kw about
prostate ca screening
40.African patient underwent infibulation….comes for antenatal
checkup….what to advise
41.48year old female underwent bilateral ovary removal…when to
prescribe HRT
42.Woman who is a mother of two children….has past h/o anorexia
with BMI 15….what to screen for her now
Option had DEXA scan which I chose
43.Patient MVA having pain chest…xray showed
15%pneumothorax….what to do
44.Pregnant lady presenting with SOB and chest pain….what to do
first….option had duplex usg of pelvic veins
45.COPD xray
46.Xraycaecal volvulus
47.Caecal carcinoma suspected…what clinical feature will you find in
patient
a. Fatigue and tiredness
b. Altered bowel habits
AAAAAAAAAAAAAAAAAAAA

48.Lady waiting for GB surgery developed hypotension and abdomen


showed rigidity and guarding ….BS reduced…what investigation to
do
49.Lat epicondylitis question
50.Patient presenting with tenderness around scrotum and
testis….whatinvstg to do first
51.Urge incontinence picture….bestinvstg
52.Old lady with osteoporosis having urge incontinence…what to
give…oxybutynin

53.Solid tr at left pole of testis….intial management


a. FNAC
b. Core biopsy
c. Orchidectomy
………….none………….US

54.19year old girl….usg showed soli/cystic tr….what is the diagnosis


55.Usg showed 1.5cm cyst….what to do…option had review after
3weeks
56.Child having pst history of viral infection….presenting with
rash….reduced platelets….diagnosis
57.Child complaing of ‘tummy aches’ just before father leaves for
work and reluctant to leave car when he reaches school but
settles in class….what will you find in history
a. Fear of losing parent
b. Nightmares
c. Bullying
AAAAAAAAAAAAAAAAAAAAAAAAAA………SEPARATION ANXIETY

58.35year old man…married presents with arthralgia…fever…


splenomegaly….blood shows atypical lymphocytes….diagnois
59.Patient having a cystic thyroid swelling…fnac showed red blood
cells and follicular cells..cyst completely collapsed and no other
symptoms…next management
a. Partial thyroidectomy
b. Repeat fnac
C.CORE

CCCCCCCCCCCCCCCCCCCCCCCCC

60.Girl treated by gp for a upper neck 3cm firm swelling as ‘tonsilitis’


presented to you….what to do next
a.fnac
b. ignore swelling
BBBBBBBBBBBBBBBBBBBBBBBBBBBB

64. guy presented with seeing dead ghost of father…father used


to live with him….when he tries to speak with the ghost it
dissolves off…what will you find in the history
a. social isolation
b.drinks full strength beer at night
c. stays at home and plays video and fantasy games

64. man waking up in the middle of night and difficulty to go back


to sleep…dx…major depression
65. cvs risk question
66. woman who is a smoker….anorexia and 15kg weight loss is
present….xray given and shows cardiomegaly…dx asked
67. pregnant lady with HSV ulcers and urinary retention….what to
give after putting in catheter
68. lady with LSIL coming for vaccination of HPV…when to give
69. obese man with complaints of loud snoring…what to do…
option had sleep study referral and cpap

71. .Middle age man with HTN, DM was on


warfarin,perindopril,statin,metformin.1wk ago he started to take
amiodarone, now present with right thigh swelling. On examination
temp 37.9, tender, red thigh. His Rt thigh is 4 cm bigger than left in
circumference. Cause of this presentation??

1.aterial embolus

2.Cellulitis

3.dvt

4.Drug reaction

CCCCCCCCCCCCCCCC

74. lady who developed chest pain…relieved on rest…what to do


next…I answered as troponin

75. lady who fell unconscious while getting up from bed….postural


hypotension present…what to do next
a. repeated postural bp checks
b. bp monitoring 24hrs
AAAAAAAAAAAAAAAAAAAAAAAAAAAAA

76. you went to visit a elderly lady who is staying alone….house


unclean….lady thinks you are her daughter and calls you by her
name…gets angry suddenly and starts getting physically violent…
what to do
a. call police
b restrain her
c.defend yourself
d. leave
DDDDDDDDDDDDDDDDDDDD
1. 77.Man ,knw case of angina. Came to take his usual prescription
from you. He said his fine just his angina is more frequent now. He
is on statin, acei , aspirin ,and some other drugs. BMI 30, smoker
40packs, total chol5.he just got divorce and eat outside food most
of the time. What to do next?
a. Increase statin
b. Increase acei
c. Asses motivation to modify risk factor
d. Arrange aecg stress test
CCCCCCCCCCCCCCCCCCCCCCC

78. postpartum lady bringing in her baby frequently…what to


assess in mother

79. Bullemia. Which Family hx is associated?


a. Depression.
b. Obesity.
c. Schizophrenia
BBBBBBBBBBBBBBBBBBBBBBBBBBBBBB

80. boy presenting with knee pain….xray features given…dx


asked…osteosarcoma
81. glaucoma eye picture…management

You might also like